SlideShare a Scribd company logo
1 of 146
Download to read offline
បជុំវ       គណិតវទ សិស ពូែកគណិតវទ ក់ទី១០
បកែ បេ យ ែកវ សិរ"

            គណិតវទ អូ   ំពិក បៃពណីេវៀត   ម 30-4   ំ 2011
បជុំវ          គណិតវទ សិស ពូែកគណិតវទ          ក់ទី១០



                 វ          អូ         ពិក បៃពណីេវ ត
                                       ំ                          មេលកទី XVII       ំ ២០១១

                         មុខ វ%ៈ គណិត វទ)*ក់ទី ១០ រយៈេពល ១៨០/ទី
                               &

 .            យ      ម             ង       មកង      នចននព

                                       x3 − 15 x2 + 78 x − 141 = 53 2 x − 9

 .     គ !ចននគ "#ជ%&ន n នង d1 < d 2 < d3 < d 4 'បន )ចកគ "#ជ%&ន * ចបផ                               ប     " n

       ក គប"ប,ចននគ "#ជ%&ន n ./ម0 ! n = d1 + d 2 + d3 + d 4
              -                         2     2    2     2


1.     2កងប3ង" គ !ម xOy នងព ចនច A                        4   2 5/កន3 ប6 " Ox នង B
                                                                       7                     4    2 5/កន3

      ប6 " Oy 89 ង, !
        7                                   : OAB            ម;     ង" O      ∆ 'ប6 "ច5< មយមន
                                                                                   7                               "

      =ម O , ) ?) ង               " =មចនចក,5 I ប
                                           -                      " AB នង        " កន3 ប6 " Ox, Oy @ងA
                                                                                         7

        ង"ប,ចនច C , D
            -                          =ង M 'ចនចក,5 ប
                                                  -                        " CD, N 'ចន ប         ពBCន OM

      នDង AB, H 'ច ,5)កង ប                      " N 2 5/ CD          ព5 ∆ ច5< , ច* ក         នចនច ប            "

      ចនច H

E.     គ ! a, b, c 'បចននព មនF#ជ%&ន ផ7GងH " a 2 + 4b 2 + 9c 2 = 14
                                        7

          យបIក"J
             %                    3b + 8c + abc ≤ 12

K.        យបIក"Jព 2011 ចននគ "#ជ%&ន,កL
             %                                                       យ គ) ង ជ/         /   ;នចននព

      ).5ផ5ប* ក Mផ5.ក ប                   "?)ចក     ច" នDង 4018

                       x2 y 2
N.     គ ! F5ប ( E ) :   +    = 1 នងប6 " ( ∆ ) : 2 x − 2 y + 4 = 0
                                      7                                               =ង B, C       @ង
                       8   4
      A'ចនច ប        ពB ប          " ( ∆ ) នង ( E ) , yB > yC O/យ A 'ចនច 2 5/ ( E ) 89 ង

      , ! ប)#ងព A P ( ∆ ) )#ងបផ                       កចនច M 2 5/ ( E ) ./ ម0 ! ប)#ងព M

       Pប6 " AB គM)#ងបផ
          7
                                                   '()&




www.keoserey.wordpress.com
                                                                                                           Page 1
បជុំវ               គណិតវទ សិស ពូែកគណិតវទ         ក់ទី១០



                                                            ចំេលយ

 .      ម      ).5 !          មម* 5នD ង

                                         (
              ( x − 5)3 = −3 x + 5 5 + 3 2 x − 9 − 9    )
      =ង       y = 5 + 3 2 x − 9 , យ/ង;ន បព<នQ                 ម

              ( x − 5)3 = −3x + 5 y − 9
              
              
              ( y − 5) = 2 x − 9
                       3
              
      .កFងRនDងFងR            ម           (1) នង (2) , យ/ង;ន

              ( x − 5)3 − ( y − 5)3 = −5 x + 5 y .

      ⇔       ( x − y )  ( x − 5)2 + ( x − 5)( y − 5) + ( y − 5)2 + 5 = 0
                                                                     
          យ ( x − 5) 2 + ( x − 5)( y − 5) + ( y − 5)2 + 5 =
                                             2
                        1          3
              =  x − 5 + ( y − 5)  + ( y − 5) 2 + 5 > 0, ∀x, y ∈ ℝ
                        2          4
       6      ម        (3)       មម*5នDង x = y

      ព (2) Sញ;ន ( x − 5)3 = 2 x − 9 ⇔                      x3 − 15 x 2 + 73x − 116 = 0

                                            x = 4
      ⇔       ( x − 4)( x − 11x + 29) = 0 ⇔ 
                         2
                                             x = 11 ± 5
                                            
                                                    2
                                                                11 − 5 11 + 5 
                                                                              
      .*ច ន        នU        ប       "       ម     ).5 !គM S = 4;     ;       
                                                               
                                                                   2      2  
 .     ឃ/ញJ x 2 ≡ 0 (mod 4) ព5 x គ*, x 2 ≡ 1 (mod 4) ព5 x

       ប/ n 'ចនន                     6 គប" ប,ចនន di
                                             -                         ទQ)          O/យ

              n ≡ d12 + d 22 + d 32 + d 4 ≡ 1 + 1 + 1 + 1 ≡ 0 (mod 4) (ក : ន ផ7យព
                                        2
                                                                                          ព )

      .*ច ន       យ/ង;ន n = 2k

       ប/ 4 ' )ចក ប              " n 6 d1 = 1 នង d 2 = 2, n ≡ 1 + 0 + d 32 + d 42 )ចកមន    ច" នDង 4 (ក :

       ន ផ7យព          ព )       .* ច ន            យ/ង;ន n )ចកមន             ច" នDង 4

      .*ច ន       {d1 , d 2 , d3 , d 4 } = {1, 2, p, q}


www.keoserey.wordpress.com
                                                                                                    Page 2
បជុំវ       គណិតវទ សិស ពូែកគណិតវទ                ក់ទី១០


       M   {d1 , d 2 , d 3 , d 4 } = {1, 2, p, 2 p}   ចZ          p, q 'ប,ចននប[ម
                                                                          -

      កងក : {d1 , d 2 , d 3 , d 4 } = {1, 2, p, q} យ/ង;ន n ≡ 3 (mod 4) (ផ7យព                                   ព )

      .*ច ន         n = 5 (1 + p 2 ) O/យ n )ចក                        ច" នDង 5 , 6   p = d3 = 5 នង n = 130

1.    •          យបIក")ផក
                    %                       ប

      =ង
                                                                                                                         O
      + P 'ចនច ប                   ពB ប     "កន3 ប6 " OI
                                                   7
                                                                                                           C1
      នDង ងBង" (C )  Dក ]                       : OCD
                                                                                                  A                      D1
                                                                                              E            N
      + J 'ចនច ប                   ពB ប     "កន3 ប6 " OI
                                                   7                                                            I             D
                                                                                                            H
      នDងប6 ")កងនD ងកន3 ប6 " Ox
           7              7                                     ង" A                                  H1                      B
                                                                                                                               F
      + E , F @ងA'ច ,5)កង ប                                     " P                                             H2
                                                                             x
                                                                                                           P
       P 5/កន3 ប6 " Ox, Oy
                 7                                                                                                                 y

      =មប^ប" យ/ង;ន

            ប/ C ≡ A , 6 ! D ≡ B ( ផ7យមក#ញ) 6 M ≡ N ≡ H ≡ I
                                   M

            ប/ C ≠ A នង D ≠ B , ព5 6 ,

      + កន3 ប6 " OI 'ប6 "ព ប
              7        7                                "ម AOB ⇒ P 'ចនចក,5ធ* CD ប
                                                                         -                                           " (C )

      ⇒ PM ⊥ CD                    ព 6 , E, M , F       4        2 5/ប6 " Simson ប
                                                                       7                          "ចនច P ច Z ∆OCD

      + កL Z កន3 ប6 " OI 'ប6 " ព ប
                   7        7                                            " AOB 6 EF ⊥ OI ⇒ EF || AB

      + AJ , EP )កង មA PនDង Ox 6                                  AJ || EP

      ព 6 =ម ទD                    -បទ= 5             យ/ង;ន
                  OJ OA ON
                    =  =   ⇒ NJ || PM ⇒ NJ ⊥ CD ⇒ N , H , J                                       "   ង"ជ
                  OP OE OM
      .*ច ន        H           4   2 5/ ងBង" (T ) Fង` " ផa              IJ       យប, កន3 ប6 " Ox, OI នងប,
                                                                                    -      7             -

      ចនច A, B 2នDង 6 I , J 2នDង ⇒ (T ) 2នD ង

      • 5ម           ប     "       នចនច

      =ង C1 , D1 @ងA'ប,ចនច
                       -                                    4    2 5/ Ox, Oy 89 ង, ! IC1 || Oy នង ID1 || Ox;

      H1 , H 2 @ងA'ចនច ប                        ពB ប   " (T ) នDង IC1 នង ID1            ព5 6 C bច                    4   2Fង` "

www.keoserey.wordpress.com
                                                                                                                              Page 3
បជុំវ        គណិតវទ សិស ពូែកគណិតវទ                          ក់ទី១០


      OC1 ) ប9 , .*ចAនD ង D bច
                c                                       4    2 ]Fង` " OD1 ) ប9 ,c                             Sញ;ន H         4    2

          5/ធ* H1 H 2 &នផ7ក I ប                " ងBង" (T ) ( 5/ក)5ងព ចនច H1 , H 2 )

      •           យបIក")ផកផ7យ
                     %

          2 5/ធ* H1 H 2 &នផ7ក I ប                  " ងBង" (T ) យ/ង dចនច H '                            =ង C ', D ' 'ចនច ប         ពB

          ប     " IH ' @ងA PនD ង Ox, Oy; P ' 'ចនច ប                                ពB ប      "កន3 ប6 " OI PនD ង ងBង" Dក
                                                                                                    7

          ] ∆OC ' D '; E′; F ′ @ងA'ច ,5)កង ប                                       " P ' P 5/ Ox, Oy; N ′ 'ចនច ប             ពB

          ប     " JH ' PនD ង AB; M ′ 'ចនចក,5 ប
                                           -                                      " C 'D'      យ/ង       e#   បfញJ O, N ', M '
                                                                                                                g

           "    ង"ជ A

      ព '.*ច ន =ង N '' 'ចនច ប                               ពB ប            " OM ' នង AB             ព5 6 =ម            ^ប" បIក"
                                                                                                                              %

      )ផក          ប យ/ង;ន
                            OJ ' OA ON ''
                                =    =     ⇒ N '' J || P ' M '
                            OP ' OE ' OM '
                   ⇒       N '' J ⊥ C ' D ' ⇒ N '', H ′, J              "    ង"ជ     ⇒ N '' ≡ N '

      .*ច ន          O, N ', M '        "   ង" ជ A (បIg            e#         យបIក")
                                                                                 %

      •         ន hន          នចនច H 'ធ* H1 H 2 &នផ7កចនច I ប                                         " ងBង" (T ) ( 5/ក)5ងព

      ចនច H1 , H 2 )

E.    #        មiព).5 !             មម* 5នD ង       6b + 16c + 2abc ≤ 24 (1)

      + Fន# -នj#            មiព Cauchy យ/ង;ន

                   6b + 16c ≤ 3(b2 + 1) + 8(c 2 + 1) = 11 + 3b 2 + 8c 2              .

                   = 11 + (a 2 + 4b 2 + 9c 2 ) − a 2 − b2 − c 2 = 25 − a 2 − b2 − c 2 .

          6      ./ ម0       យបIក" (1) យ/ង Aន")
                                %                                            e#           យបIក"#
                                                                                             %                មiព

                   a 2 + b2 + c 2 − 1 ≥ 2abc (2) , ច Z a, b, c មនF#ជ%&ន

      + =មប^ប" a 2 + 4b2 + 9c 2 = 14 , #                    មiព (2) bច                           k/ង#ញ.* ច          ង    ម

                  14(a 2 + b2 + c 2 ) − (a 2 + 4b2 + 9c 2 ) ≥ 28abc .

               ⇔ 13a 2 + 10b 2 + 5c 2 ≥ 28abc .

               ⇔ (13a 2 + 10b 2 + 5c 2 ) a 2 + 4b 2 + 9c 2 ≥ 28 14abc

www.keoserey.wordpress.com
                                                                                                                             Page 4
បជុំវ        គណិតវទ សិស ពូែកគណិតវទ                                 ក់ទី១០


      + Fន# -នj#          មiព Cauchy ម-ង ទ@                   ច Z 28 ចនន យ/ង;ន

                13a 2 + 10b 2 + 5c 2 ≥ 2828 ( a 2 )13 (b 2 )10 (c 2 )5 = 2814 a13b10 c 5

                                                                      (                   )
                                                                                              2
      នង         a 2 + 4b 2 + 9c 2 ≥ 1414 a 2 (b 2 ) 4 (c 2 )9 = 14       14
                                                                               ab 4 c 9


                                                                                                           (                   )
                                                                                                                                    2
      + .* ច 6          (13a 2 + 10b 2 + 5c 2 ) a 2 + 4b 2 + 9c 2 ≥ 2814 a13b10 c 5 14                         14
                                                                                                                    ab 4 c 9            = 28 14abc

          មiព ក/ &ន ព5 (a, b, c) = (1,1,1)

      5l "           e#;ន          យបIក" ច^5"
                                      %

K.     ព5)ចកចននគ "#ជ%&ន,មយ នD ង 4018 6 ប,-                                                             :5"              e#      4        2កង        ន

      {0, 1, ..., 4017}
      កងប,-             :5"         ង 5/ យ/ង)ចក ចញ' កmម.* ច                                        ង       ម

      + កmមទមយ ម&នប,ចនន ព5)ចកនD ង 4018 ;ន
                    -                                                                                  :5"                   /n 0

      + កmមទព            ម&នប,ចនន ព5)ចកនD ង 4018 ;ន
                              -                                                                        :5"             /n 1 M            /n 4017

      + កmមទប ម&នប,ចនន ព5)ចកនD ង 4018 ;ន
                   -                                                                                :5"               /n 2 M             /n 4016

      ............

      + កmមទ 2009 ម&នប,ចនន ព5)ចកនD ង 4018 ;ន
                       -                                                                                   :5"                 /n 2008 M           /n 2010

      + កmមទ 2010 ម&នប,ចនន ព5)ចកនD ង 4018 ;ន
                       -                                                                                   :5"                 /n 2009

      .*ច ន &នSងF                      " 2010 កmម, ) )ប '&ន 2011 ចនន 6 =ម ទD                                                            -បទ Dirichlet

       ?ងពក?             e#&នព ចនន ).5                  :5"កង ប&:#ធ)ចកនD ង 4018 oក"ច*5កង កmម
                                                                                 3

      'មយA

       ន គM'ព ចនន).5                     e# ក     Z ប/ ព ចនន ន &ន                                   :5"               /nA 6 ផ5                     ងប        "

      ពក?)ចក            ច"នDង 4018 , ប/ ពក?&ន                  :5" ផpងA 6 ផ5ប* ក ប                                                      "ពក?)ចក         ច"

      នDង 4018

N.    =ង A(2 2 sin t ; 2 cos t ) ∈ ( E ), t ∈ [ 0; 2π ]

                                                                        π
                                                               4 2 sin  t −  + 4
                                        4sin t − 4 cos t + 4            4
       យ/ង&ន              d ( A, (∆)) =                      =
                                                  6                       6


www.keoserey.wordpress.com
                                                                                                                                                      Page 5
បជុំវ          គណិតវទ សិស ពូែកគណិតវទ                ក់ទី១០


      .*ច ន     d ( A;(∆)) ធបផ              ព5
                     π            3π
              sin(t − ) = 1 ⇒ t =    ⇒ A(2; − 2)
                     4             4
                                                     t = 0 ⇒ B(0; 2)
                                     π        1
              d ( A; (∆)) = 0 ⇒ sin  t −  = −    ⇒  3π
                                        4      2   t =    ⇒ C (−2 2; 0)
                                                          2
         ម               (            )
                ( AB ) : 2 + 2 x + 2 y − 4 = 0

                                                                                        π       π
              d ( M , ( AB )) =
                                  (              )
                                      2 + 2 2 2 sin t + 4 cos t − 4
                                                                           =
                                                                               8 2 sin  t +  cos − 4
                                                                                        8       8
                                                     10 + 4 2                         10 + 4 2
      .*ច ន     d ( M , ( AB)) )#ងបផ                 ព5
                     π             11π              3π        3π
              sin(t + ) = −1 ⇒ t =     ⇒ M (−2 2 sin ; − 2 cos )
                     8              8                8         8
                                                          '()&



                   វ1        េសរ បឡងអូ                    ពិចេវ ត
                                                          ំ               ម*ក់ទី១០ េលកទី XVII

                                                           វ        ទី១

 .              យ      ម              ង          ម 5/      នចននព                          (
                                                                           9 x3 + 8 = 2 x 2 + 8     )   (1)

 .    បfញJ មន&នប,ចននគ " x, y, z ផ7GងH " ទ6ក"ទនង
        g        -                   7

                 ( x + 2010)2 + ( x + 2012)2 = ( x + y + z + 2008)( y + z − x − 2014)
1.     គ !         : ABC  Dកកង ងBង" (O) នង ក ] ងBង" ( I )
                                             D                                                =ង D , E , F        @ង

      A'ប,ចនចប9 ប
          -                                " ( I ) PនD ងប, ជmង BC , CA, AB
                                                          -                                    ង" ងBង" (O1 ) ប9

        ]នDង ( I )      ង"ចនច D O/យប9 កងនDង (O)                            ង"ចនច K , ងBង" (O2 ) ប9            ]នD ង

      (I )    ង" ចនច E O/យប9                 ]នD ង (O)          ង" M , ងBង" (O3 ) ប9      ]នD ង ( I )         ង" F

       O/យប9 កងនD ង (O)                   ង" ចនច N         បfញJ
                                                             g

      a ). ប,ប6 " DK , EM , FN
             - 7                                          "A    ង"ចនច P

      b). ប6 " OP
            7                  "=មF          *       ង" H ប     "         : DEF

E.     គ ! a, b, c 'ចននព មនF#ជ%&នប).5 ផ7GងH " a 2 + 4b 2 + 9c 2 = 14
                                           7

www.keoserey.wordpress.com
                                                                                                                       Page 6
បជុំវ       គណិតវទ សិស ពូែកគណិតវទ                 ក់ទី១០


      បfញJ
        g            3b + 8c + abc ≤ 12

                                       2011                     2
K.     គ !Fនគមនj F ( x) =               ∑ (k − 2011x) C2011 xk (1 − x)2011−k
                                                       k

                                        k =0

       ក C5ធបផ            ប       "Fនគមនj 2 5/ច 63                  [0;1]
                                                          '()&


                                                                ចំេលយ

 .    5កq: x ≥ −2
          r

        ម           មម*5នDង            9 ( x + 2)( x 2 − 2 x + 4) = 2  2( x + 2) + x 2 − 2 x + 4 (2)
                                                                                                
            យ x 2 − 2 x + 4 = ( x − 1) 2 + 3 ≥ 3

      )ចកFងRSងព Cន                ម        (2) នDង x 2 − 2 x + 4 , យ/ង;ន

                  x+2             x+2
                4 2          −9 2          +2=0
                  x − 2x + 4    x − 2x + 4
                   x+2               1
      ⇒                         =      ⇒ x = 9 ± 109 (យក)
                x2 − 2 x + 4        16
 .      ម       ).5 !           មម* 5នD ង

                ( x + 2010 )2 + ( x + 2012 )2 = ( y + z − 3)2 − ( x + 2011)2
      ⇔         ( x + 2010 )2 + ( x + 2011)2 + ( x + 2012 )2 = ( y + z − 3)2
      ⇔         3 x 2 + 12066 x + 20102 + 20112 + 20122 = ( y + z − 3) 2
      FងR     ង ឆBង ប      "      ម       )ចកនD ង 3 ;ន              :5"        /n 2 , FងR   ង   - ប   "   ម

      )ចកនDង 3 ;ន          :5"                 /n 1 M    /n 0

      .*ច ន         ម     ).5 !AនU
                                n                       'ចននគ "

1.    a ). .ប*ង យ/ង             យបIក" Lemma : " ! X , Y 'ព ចនច 2 5/ ងBង" (O) , ងBង" (O ')
                                   %

      មយ ប9 នDង XY              ង" U O/យប9 កងនD ង (O)                   ង" V       ព5 6 , ប6 " UV
                                                                                            7                 " =ម

      ចនចក,5 Z ប
           -                           "ធ* XY មន&នផ7ក V "


www.keoserey.wordpress.com
                                                                                                                Page 7
បជុំវ        គណិតវទ សិស ពូែកគណិតវទ                ក់ទី១០


      ព '.*ច ន , ពន !ចtប" ប)5ងងផa                         V : (O ') → (O)           ព5 6       XY → d

       ផ7GងH " d || XY O/យ d ប9 នD ង (O)
            7                                             ង" Z ' បiព ប
                                                                 *                   " U ⇒ Z 'ចនចក,5
                                                                                                   -

       ប   "ធ* XY
                                                                               A A1
                                   V
                              O'                                                                       B0
                                       B
                                       Y                       C0
                                                                                                            M
           X                                                            O3             O
                      O                                        N                            E    O2
                                                                                      I                 C1
                                                                   B1            D
                          Z                                             B                          C
                                                                                 O1
                                                                                   K       A0
      ពន !5l ").5 !

      =ង A , B1 , C1 'ចនច ប
          1                                ពB ប    " DK , EM , FN PនD ង (O) , =ម Lemma យ/ង

      ;ន A , B1 , C1 'ប,ចនចក,5 ប
          1             -    -                              "ធ* BAC , CBA, ACB             =ង A0 , B0 , C0

      'ប,ចនចឆ3 ប
         -                         " A , B1 , C1 ធ@បនDង O , ព5 6 ∆A0 B0C0 , ∆A B1C1 &ន
                                      1                                       1

      ប, ជmង
        -            បA

      មu9 ង ទ@       B0C0 , EF ⊥ AI ; A0C0 , FD ⊥ BI ; A0 B0 , DE ⊥ CI

      Sញ;ន ∆A0 B0C0 , ∆DEF &នប, ជmង
                               -                               បA           .* ច 6 ∆A B1C1 , ∆DEF
                                                                                     1

      &នប, ជmង
          -                បA O/យមន                 /A ( ∆A B1C1  Dកកង (O), ∆DEF  Dកកង ( I ) )
                                                     n     1

      ⇒ ∃ ចtប"ប)5ងង ប)5ង ∆DEF P' ∆A1B1C1

      ⇒ DA1 , EB1 , FC1 ប          ពBA      ង"ផa     P ប    "ចtប" ប)5ងង

      b). ព     ន a). ⇒ P, O, I        "    ង" ជ A (1)

      =ង A ', B ', C ' 'ប,ចនច ប
                          -                        ពB ប    "ប,កព
                                                              -             " DD ', EE ', FF ' ប       " ∆DEF

      នDង (1)        យ EE ' FF '  កកង 6 B ′A′D = B ′ED = C ′FD = C ′A′D ⇒ D '
                                   D

      ចនចក,5 ប
           -                  "ធ* B ' C ' ⇒ B ′C ′ ⊥ ID ⇒ B ′C || BC ( Z             BC ⊥ ID )

      .*ចA).        C ' A ' || CA, A ' B ' || AB , O/យ H 'ផa            ងBង" កកង ∆A′B ′C ′
                                                                              D

      ព 6 ∆ABC , ∆A′B′C ′ &នប, ជmង
                              -                            បA នងមន              /nA ( Z ∆ABC  Dក


www.keoserey.wordpress.com
                                                                                                                Page 8
បជុំវ       គណិតវទ សិស ពូែកគណិតវទ                   ក់ទី១០


      កង (O), ∆A′B ′C '  Dកកង ( I ) ) ⇒ ∃ ចtប" ប)5ងង ប)5ង ∆ABC                                                    P'

      ∆A′B ′C ′

      6 ! AA ', BB ', CC ' ប          ពBA     ង" ផa ប)5ងង Q ⇒ Q , I , O                                     "    ង" ជ A (2)

      មu9 ង ទ@ , I , H 'ផa  Dកកង ∆ABC , ∆A′B′C ′ ⇒ Q, H , I                                     "   ង" ជ A (3)

      ព (1), (2), (3) យ/ង;នបIg              e#;ន        យបIក"
                                                           %

E.    #     មiព).5 !          មម* 5នD ង       6b + 16c + 2abc ≤ 24 (1)

      + Fន# -នj#      មiពក*           យ/ង;ន

               6b + 16c ≤ 3(b 2 + 1) + 8(c 2 + 1) = 11 + 3b 2 + 8c 2

               = 11 + (a 2 + 4b 2 + 9c 2 ) − a 2 − b 2 − c 2 = 25 − a 2 − b 2 − c 2
          6 ./ ម0      យបIក" (1) យ/ង Aន")
                          %                                  e#              យបIក"#
                                                                                %                        មiព

               a 2 + b 2 + c 2 − 1 ≥ 2abc (2) , ច Z a, b, c មនF#ជ%&ន

      + =មប^ប" a 2 + 4b 2 + 9c 2 = 14 ,#           មiព (2) bច                                   k/ង#ញ.* ច              ង       ម

                  (                 ) (                 )
              14 a 2 + b 2 + c 2 − a 2 + 4b 2 + 9c 2 ≥ 28abc

      ⇔        13a 2 + 10b 2 + 5c 2 ≥ 28abc

      ⇔        (13a2 + 10b2 + 5c2 )       a 2 + 4b 2 + 9c 2 ≥ 28 14abc

      + Fន# -នj#      មiពក*          ម-ង ទ@ ច Z 28 ចនន យ/ង;ន


                                              ( ) (b2 ) ( c2 )
                                                   13        20          5          14
              13a 2 + 10b 2 + 5c 2 ≥ 2828 a 2                                = 28 a13b10c5


                                             ( ) ( c2 )             (                   )
                                                  4      9                                  2
                                                                        14
      នង       a 2 + 4b 2 + 9c 2 ≥ 1414 a 2 b 2              = 14            ab 4c9

      + .*ច 6


      (13a2 + 10b2 + 5c2 )                                                     (                     )
                                                                                                         2
                                   a 2 + b 2 + c 2 ≥ 2814 a13b10c5 14              14
                                                                                        ab 4 c 9                 = 28 14abc

      #     មiព ក/ &ន ព5 (a, b, c) = (1,1,1) , 5l "                           e#;ន                   យបIក"
                                                                                                        %
                             n
K.        យ/ង&ន        A=   ∑ (k − nx)2Cnk xk (1 − x)n−k
                            k =0


www.keoserey.wordpress.com
                                                                                                                                   Page 9
បជុំវ                គណិតវទ សិស ពូែកគណិតវទ                          ក់ទី១០


                            n                                      n                                            n
              = (nx)   2
                           ∑      Cn x k (1 − x)n − k
                                   k
                                                                 +∑k         2
                                                                                 Cn x k (1 − x) n −k
                                                                                  k
                                                                                                         − 2nx ∑ kCn x k (1 − x)n − k
                                                                                                                   k

                           k =0                                   k =0                                         k =0
                                n                                        n
      ពន !        A1 =       ∑ kCnk xk (1 − x)n−k = ∑ kCnk xk (1 − x)n−k
                             k =0                                      k =1
                                    n                                                 n
                           = n∑           Cn −1 x k (1 − x)n −k
                                           k −1
                                                                       = nx ∑ Cn −1 (1 − x)n − k
                                                                               k −1

                                  k =1                                               k =1
                                                         n−k
                           = nx [ x + (1 − x) ]                  = nx
                                  n                                              n
                   A2 =         ∑k          2
                                                Cn x k (1 − x)n − k
                                                 k
                                                                         = ∑ k 2Cn x k (1 − x)n − k
                                                                                 k

                                k =0                                         k =1
                                      n
                            = n ∑ Cn −1 x k (1 − x)n − k
                                   k −1

                                    k =1
                                     n                                               n
                            = n ∑ Cn −1 x k (1 − x)n − k + n ∑ (k − 1)Cn −1 x k (1 − x) n − k
                                   k −1                                k −1

                                    k =1                                          k =1
                                                 n
                            = nx + n ∑ (k − 1)Cn −1 x k (1 − x) n − k
                                               k −1

                                                k =1
                                                         n
                            = nx + n(n − 1) ∑ Cn − 2 x k (1 − x)n − k = nx + n(n − 1) x 2
                                               k −2

                                                        k =2
                                        n
                                      ∑ Cnk xk (1 − x)n−k = [ x + (1 − x)]
                                                                                                n
                           A3 =                                                                     =1
                                    k =0

      .*ច ន     A = (nx )2 + nx + n(n − 1) x 2 − 2(nx )2 = nx(1 − x)

      Fន# -នj5ទQផ5                  ង 5/ យ/ង;ន                     f ( x ) = 2011x(1 − x)

         យ x ∈ [ 0;1] 6                     x,1 − x ≥ 0          ព 6 =ម#                       មiពក*
                                                             2
                              x + (1 − x)    2011                                                                           1
              f ( x) ≤ 2011.               =      ,                                     Iw    មiព ក/ &ន ព5 x =
                                   2          4                                                                             2
                                            2011              1
      .*ច ន     max f ( x) =                     ទទ5;ន ព5 x =
                                             4                2
                                                                 '()&




www.keoserey.wordpress.com
                                                                                                                                    Page 10
បជុំវ      គណិតវទ សិស ពូែកគណិតវទ                ក់ទី១០



                    វ1     េសរ បឡងអូ          ពិចេវ ត
                                              ំ                 ម*ក់ទី១០ េលកទី XVII

                                                វ         ទី២
                                         x + y + 1 + 1 = 4( x + y )2 + 3. x + y
                                        
 .              យ បព<នQ        ម        
                                        30 x − 4 y = 2011
                                        
 .     ក គប"ប,ចនន).5&ន 5ខបខ7ង")ចក
              -                                              ច" នDង 11 89 ង, !ផ5)ចក ប                  "?

           /nនDងផ5ប*ក          ប    "ប, 5ខ=មខ7ង" ប
                                       -                         "?

1.     គ !          : ABC ).5 A 'ម * ចបផ ,  កកង ងBង" (O), ចនច D ច5<
                                             D                                                     2 5/

      ធ* * ច BC       ម.uទ< ប         " AB នង AC           " AD @ងA            ង" E នង F       =ង T '

      ចនច ប         ពB ប   " BE នង CF       ប6 "
                                              7           "=ម T O/យ                 បនD ង AB    " AD        ង"

      N        ង" ប 5k*            ម TNDM     =ង P 'ចនចក,5 ប
                                                         -                            " MC O/យ I 'ចនច

       ប     ពB ប    " PT នD ង ម.uទ< ប        " OP

           យបIក"J I ច5<
              %                       2 5/)ខpនD ងមយ

                                                                                    2 1      4
E.     គ ! x, y, z 'ប,ចននព #ជ%&ន ផ7GងH " ប,5កq:
                      -               7    -   r                                     + ≤ 1 នង + y ≤ 2 ,
                                                                                    x y      z
       ក C5 *ចបផ           ប       "ក នyម   P ( x, y , z ) = x + 9 y + z

K.         យបIក"J កង 17 ចននគ " ,កL
              %                                           យ ) ង&ន 9 ចនន).5&នផ5ប* ក

      )ចក     ច"នDង 9

N.    ប,កព*5 ប
        -                  " ∆ABC 'ចនច).5&នក* F                       ន'ចននគ " O/យមន&ន

             :, * ច'ង ∆ABC នង&ន^ង.*ចនDង ∆ABC ទ ច Z កព*5 ប                                         "'

      ប,ចនច&នក* F
        -                            ន'ចននគ "        បfញJ ផa
                                                       g                    D ប      " ងBង" ក ] ប
                                                                                             D          "

      ∆ABC មន)មន'ចនចមយ).5&នក*F                               ន'ចននគ " ទ
                                               '()&




www.keoserey.wordpress.com
                                                                                                            Page 11
បជុំវ      គណិតវទ សិស ពូែកគណិតវទ           ក់ទី១០



                                                  ចំេលយ

                                        x + y + 1 + 1 = 4( x + y )2 + 3. x + y
                                       
 .             យ បព<នQ        ម                                                       (1)
                                       30 x − 4 y = 2011
                                       
      =ង u =       x+ y ≥0

             u 2 + 1 + 1 = 4u 4 + 3u (2)
            
            
      (1) ⇔ 30u 2 − 34 y = 2011
            
            x + y = u
                       2
            

      (2) ⇔        u 2 + 1 + 1 = 4u 4 + 3u ⇔            u 2 + 1 − 3u + 1 − 4 y 4 = 0

                   u 2 + 1 − 3u 2
           ⇔                        + (1 − 2u 2 )(1 + 2u 2 ) = 0
                    u + 1 + 3u
                     2

                                1                 
           ⇔ (1 − 2u 2 )               + 1 + 2u 2  = 0
                          u 2 + 1 + 3u
                                                  
                                                   
           ⇔ 1 − 2u 2 = 0 ⇔ 1 − 2( x + y ) = 0
                    2 x + 2 y = 1          2013      998
      Sញ;ន                            ⇔ x=      ;y=−
                    30 x − 4 y = 2011       34       17
                                           2013      998
      .*ច ន    U     ប    " បព< នQគM x =        ;y=−
                                            34       17
 .    =ង x = abc 'ចនន).5                e# ក

                    100a + 10b + c = 11(a 2 + b 2 + c 2 ) (1)
                    
                    0 ≤ a; b; c ≤ 9
       យ/ង;ន        
                    a ≠ 0
                     a; b; c ∈ ℕ
                    
       យ/ង&ន        (1) ⇔ 99a + 11b + ( a + c − b) = 11( a 2 + b 2 + c 2 )

              ⇔ a + c − b = 11k )          −8 ≤ a + c − b ≤ 18 ⇒ k = 0 M k = 1

      + k = 0 ⇒ a +c−b = 0 ⇒ b = a +c




www.keoserey.wordpress.com
                                                                                             Page 12
បជុំវ     គណិតវទ សិស ពូែកគណិតវទ       ក់ទី១០


      (1) ⇔ 9a + b = a 2 + b2 + c 2 ⇔ 9a + a + c = a 2 + (a + c)2 + c 2
          ⇔ 10a + c = 2(a 2 + ac + c 2 ) ⇔ c = 2n (n ∈ ℕ)

          ⇔ 10a + 2n = 2( a 2 + 2na + 4n 2 ) ⇔ a 2 + (2n − 5) a + 4n 2 − n = 0

      5កq:r     ∆ ≥ 0 ⇒ − 12n 2 − 16n + 25 ≥ 0

                −4 − 91     −4 + 91
           ⇒            ≤n≤         ⇒ n=0               ⇒ c = 0; b = a នង a 2 − 5a = 0
                    6           6
           ⇒ a = b = 5 នង c = 0 ⇒ x = 550

      + k = 1 , យ/ង;ន a + c − b = 11 ⇒ b = a + c − 11 6

      (1) ⇔ 99a + 11b + 11 = 11( a 2 + b 2 + c 2 )

           ⇔ 9a + b + 1 = a 2 + b2 + c 2 ⇔ 9a + a + c − 11 + 1 = a 2 + (a + c − 11) 2 + c 2
           ⇔ 10a + c − 10 = 2a 2 + 2c 2 + 121 + 2ac − 22a − 22c

           ⇔ 2a 2 + 2c 2 + 2ac − 32a + 23c + 131 = 0 ⇒ c 'ចនន

           ⇔ c = 2n + 1 .

      .*ច 6 យ/ង;ន a 2 + (2n − 15)a + 4n 2 − 19n + 55 = 0

       យ/ង&ន ∆ = −12n 2 + 16n + 25 ≥ 0

           4 − 91     4 + 91
      ⇒           ≤n≤        ⇒ n ∈ {0; 1; 2}
              6          6
      • n = 2 ⇒ c = 5; b = a − 6 នង a 2 − 11a + 33 = 0 (AនU
                                                         n             )

      • n = 1 ⇒ c = 3; b = a − 8 នង a 2 − 13a + 40 = 0

               ⇒ a = 8; b = 0 នង c = 3 ⇒ x = 803
                                                                                   A
      • n = 0 ⇒ c = 1; b = a − 10 < 0
      .*ច ន    ប,ចនន).5
                 -                 e# កគM x = 550 នង x = 803                           N
                                                                           K           E O
1.    + =ង H 'ចនច ប             ពBទព ប    " BE នDង ងBង" ( ABC )
                                                                                   T
                                                                                       F
              K 'ចនច ប          ពBទព ប   " CF នDង ងBង" ( ABC )                             I
                                                                                                     C
       យ/ង;ន      AD = CK នង AD = BH                                           B
                                                                                           D   P
      ⇒ BKHC 'ច                 :Zយ      ម;
                                                                                       M
www.keoserey.wordpress.com
                                                                                                   Page 13
បជុំវ       គណិតវទ សិស ពូែកគណិតវទ        ក់ទី១០


      ⇒ TB = TK ⇒ TB + TC = TK + TC = CK = AD .

      + យ/ង&ន TB = NA ⇒ ND = TC )                          ND = TM ⇒ TC = TM

      ⇒ ∆TMC              ម;          ង" T

                      1800 − MTC 1800 − AFT
      ⇒ TCM =                   =           = OFC
                           2          2
      ⇒ CM || OF .

      ⇒ M         4   2 5/ប6 " (∆ )
                            7                     " =ម C O/យ)កងនD ង AC 2នDង

      ⇒ P ច5<             2 5/ (∆ ) 2នD ង

      +     យ I       4    2 5/ ម.uទ< ប              " OP 6 IO = IP )     IP = d ( I ; ∆ ) ( Z IP

      )កងនDង (∆ ) )

      Sញ;ន I ) ង               4     2 5/;9^9 ប* 5).5&នកន'ចនច O O/យប6 " ;ប" ទ
                                                                     7

      គM (∆ ) , ព5 D ច5<               2 5/ធ* *ច BC

                                                        2 1 1
E.         យ x, y , z #ជ%&ន 6 យ/ង;ន 1 ≥                  + >  ⇒ y > 1 ⇒ y ∈ (1; 2)
                                                        x y y
                          2    1 y −1         2y         2
                            ≤1− =      ⇒ x≥       = 2+
                          x     y  y         y −1      y −1
       យ/ង&ន
                          4              4
                            ≤2− y ⇒ z ≥
                          z             2− y
                                       2    4
      .*ច 6    P ( x, y , z ) ≥          +      + 9y + 2
                                     y −1 2 − y
                                   1                   4
                               = 2      + 9( y − 1)  +    + 9(2 − y ) + 2 ≥ 26
                                   y −1              2− y
                                              2y        4    1
          I " = " ក/ &ន ⇔ x =
           w                                      ;z=      ;     = 9( y − 1)
                                             y −1     2 − y y −1
            4                           4
      នង        = 9(2 − y ) ⇔ x = 8, y = នង z = 6
           2− y                         3
K.    + យ/ង           យបIក" Lemma
                         %                       ង     ម

      " កង 5 ចននគ ",កL                   យ, ) ង&នបចនន).5&នផ5ប*ក)ចក                   ច" នDង 3 "

          ^យ

www.keoserey.wordpress.com
                                                                                                    Page 14
បជុំវ       គណិតវទ សិស ពូែកគណិតវទ         ក់ទី១០


      ក :ទមយ កង 5 ចននគ "                   ង 5/, &នបចនន k/ង ).5&ន                   :5" មA ព5

      )ចកនDង 3 6 Lemma                ង 5/គMព

      ក :ទព       កង 5 ចននគ "            ង 5/, &ន ច'ងបចនន).5&ន                       :5" មA ព5

      )ចកនDង 3 6 នD ង&នបចនន).5&ន                       :5" @ងAគM 0;1; 2 ព5)ចកនDង 3

       ព5 6 ផ5ប* កCនបចនន ន )ចក                    ច" នDង 3    Sញ;ន Lemma ព

      .*ច ន   Lemma          e#;ន        យបIក"
                                            %

      + Fន# -នj Lemma             ង 5/ យ/ង;ន

      • យក 5 កង 17 ចននគ ").5 ! 6 កង 5 ចនន).5 e#;នយក &នបចនន).5
      &នផ5ប*ក)ចក            ច" នDង 3     =ង បព<នQបចនន ន              យ (a1 ; b1 ; c1 ) នង=ង

      m1 = a1 + b1 + c1

      • យក 5 កង 14 ចននគ " 2 5" 6 កង 5 ចនន).5 e#;នយក &នបចនន).5
      &នផ5ប*ក)ចក            ច" នDង 3     =ង បព<នQបចនន ន              យ (a2 ; b2 ; c2 ) នង=ង

      m2 = a2 + b2 + c2

      • យក 5 កង 11 ចននគ " 2 5" 6 កង 5 ចនន).5 e#;នយក ន &នបចនន
      ).5&នផ5ប* ក)ចក                ច" នDង 3   =ង បព<នQបចនន ន                យ (a3 ; b3 ; c3 ) នង

      =ង m3 = a3 + b3 + c3

      • យក 5 កង 8 ចននគ " 2               5" 6 កង 5 ចនន).5               e#;នយក ន &នបចនន

      ).5&នផ5ប* ក)ចក                ច" នDង 3   =ង បព< នQបចនន ន            យ (a4 ; b4 ; c4 ) នង=ង

      m4 = a4 + b4 + c4

      • កង 5 ចននគ " 2              5", &នបចនន).5&នផ5ប* ក)ចក                     ច" នDង 3     =ង បព< នQ

      បចនន ន         យ (a5 ; b5 ; c5 ) នង=ង m5 = a5 + b5 + c5

      កង 5 ចននគ " m1; m2 ; m3 ; m4 ; m5 &នបចនន).5&នផ5ប* ក)ចក                               ច" នDង 3

      zប&J បចនន 6 គM mi ; m j ; mk               ព5 6 9 ចនន a1 ; bi ; ci ; a j ; b j ; c j ; ak ; bk ; ck

      &នផ5ប*ក)ចក            ច" នDង 9 (បIg       e#;ន      យបIក")
                                                             %




www.keoserey.wordpress.com
                                                                                                       Page 15
បជុំវ        គណិតវទ សិស ពូែកគណិតវទ          ក់ទី១០


N.        យ/ងbច ជ/           /   បព<នQក*F          ន Oxy 89 ង, ! A, B , C &នក* F            ន

      A(0; 0), B(a; b), C (c; d ) ).5 a; b; c; d 'ប,ចននគ "
                                                    -

      zប&J D( x, y ) 'ផa             ប      " ងBង" ( ABC ), D 'ចនច&នក*F            ន'ចននគ "

          យ/ង;ន AD 2 = BD 2              ⇒ x 2 + y 2 = ( x − a ) 2 + ( y − b) 2

                                         ⇒ a 2 + b 2 − 2ax − 2by = 0

      .*ច 6 a 2 + b 2 'ចននគ* ⇒ a, b &ន5កq: គ*                              .* ចA ⇒ a + b នង a − b

      'ចននគ*

      .*ចA). , c + d , c − d 'ចននគ*

             ab ( a + b ) 2 − ( a − b ) 2
      ⇒         =                         'ចននគ "
              2             4
                                          a +b a −b        c+d c−d 
      .*ច ន ប,ចនច X = 
              -                                ;     នង Y =    ;     'ប,ចនច&ន
                                                                           -
                                          2      2          2   2 
      ក*F            ន'ចននគ "

                       ( a + b) 2 ( a − b) 2 a 2 + b 2 AB 2                        AC 2
      )     AX 2 =               +          =         =     , .* ចA). យ/ង&ន AY 2 =
                            4          4         2      2                           2
                                     2                      2
                a+b−c−d   a−b−c+ d 
          XY = 
             2
                          +         
                   2         2     
                  1
                    ( (b − c) + (a − d ) ) + ( (a + d ) − (b + c) ) 
                                           2                        2
                 =
                  4                                                 
                  1
                 = (b − c)2 + (a − d )2 + 2(b − c)(a − d ) + (a + d )2 + (b + c)2 − 2(a + d )(b + c) 
                  4                                                                                  
                  1
                 =  2a 2 + 2d 2 + 2b 2 + 2c 2 + 2ab − 2ac − 2bd + 2cd − 2a − 2ac − 2bd − 2cd 
                  4                                                                               
                     1 2
                 =       2a + 2b 2 + 2c 2 + 2d 2 − 4ac − 4bd 
                     4                                      
                   (a − c) 2 + (b − d ) 2 BC 2
                 =                       =
                             2             2
      6 ! ∆AXY &ន^ង.*ចនD ង ∆ABC នង AXY *ច'ង ∆ABC , ផ7យពប^ប"

      .*ច ន          D មនbច'ចនច).5&នក*F                         ន'ចននគ " ទ
                                                   '()&

www.keoserey.wordpress.com
                                                                                                    Page 16
បជុំវ           គណិតវទ សិស ពូែកគណិតវទ       ក់ទី១០



                   វ1     េសរ បឡងអូ              ពិចេវ ត
                                                 ំ            ម*ក់ទី១០ េលកទី XVII

                                                  វ        ទី៣
                                                        x 2 + 4 y 2 − 4 x + 12 y + 11 = 0
                                                       
 .              យ បព<នQ      ម               ង   ម      2
                                                        x + 4 y − 2 xy − x + 4 y − 12 = 0
                                                                   2
                                                       
 .     កU   'ចននគ " ប            "       ម       x1 + x2 + ... + x15 = 1215
                                                  4    4          4


1.          : ABC ផ7GងH "5កq:FB ប/ ? ផ7GងH " 5កq:
                       7     r            7      r                            ង   ម

                                         cos A cos B cos C 17
                                              +     +     =
                                           8    15    17    120
E.     គ ! a, b, c 'បចននព #ជ%&ន ផ7GងH " abc = 1
                                     7                              បfញJ
                                                                      g

                                  1 1 1  1    1    1  9
                                  + +       +    +     ≥
                                  a b c 1+ a 1+ b 1+ c  2
K.    កង    ន T = {1, 2, ..., 2010}          /&នប9 6នចនន).5)ចកមន
                                                    n                         ច" នDង 2,3,5, 7,11 ?

N.     គ !ច        :)កង ABCD &ន M 'ចនចក,5 ប
                                        -                                 " AB , N     4   2 5/កន3

      ប6 "ព ប
        7             " BCD      =ង P 'ច ,5)កង ប                   " N 2 5/ BC

           យបIក"J ប/ MN ⊥ DP 6
              %                                            : AND       ម;
                                                 '()&


                                                      ចំេលយ

 .     បព<នQ).5 !       មម* 5នDង

                                                     x 2 + 4 y 2 − 4 x + 12 y + 11 = 0 (1)
               x 2 + 4 y 2 − 4 x + 12 y + 11 = 0
                                                   
                                                 ⇔       3x − 23
              (2 x + 8) y = 3x − 23
                                                   y =                 (2)
                                                          2x + 8
      ជន      (2) ច*5 (1) យ/ង;ន
                                     2
                    3x − 23              3x − 23
              x + 4
               2
                              − 4 x + 12.         + 11 = 0
                    2x + 8               2x + 8
      ⇔       4 x 4 + 16 x3 + 88 x 2 − 720 x + 620 = 0
                                                         x =1
      ⇔       ( x − 1)( x − 4)(4 x 2 + 32 x + 204) = 0 ⇔ 
                                                         x = 4
www.keoserey.wordpress.com
                                                                                                     Page 17
បជុំវ       គណិតវទ សិស ពូែកគណិតវទ                ក់ទី១០


      + ច Z x =1 ⇒                 y = −2

      +ចZ         x = 3 ⇒ y = −1

      .*ច ន        ម          &នច 5/យព (1; −2), (3; −1)

 .        កU    'ចននគ " ប               "   ម         x1 + x2 + ... + x15 = 1215
                                                       4    4          4


      +ចZ         xi = 2k : xi4 = 16k 4 ≡ 0 (mod 16)

      +ចZ         xi = 2k + 1 , យ/ង;ន                xi4 − 1 = ( xi − 1)( xi + 1)( xi2 + 1) = 4k ( k + 1)( xi2 + 1)

            យ k ( k + 1) ⋮ 2; xi2 + 1 ⋮ 2 ( Z            xi         ) 6       xi4 − 1⋮ 16 M xi4 ≡ 1 (mod 16)
                       15
          យ/ង;ន      ∑ xi4 ≡ r (mod 16) កង 6                     0 ≤ r ≤ 15
                       i =1

      មu9 ង ទ@      1215 = 75.16 + 15 ≡ 15 (mod 16) , .*ច 6 xi                            ច Z គប" i = 1,15

          O/យ     យ 7 4 = 2401 > 1215 6                       xi ≤ 5 ∀i

            យ 54 + 54 = 1250 > 1215 6 &ន ច/នបផ ចនន xi មយ, zប&JគM x15 ,

          ផ7GងH " x15 = 54 = 625
               7   4


          ព5 6      x1 + x2 + ... + x14 = 1215 − 625 = 590 នង xi ≤ 3 ∀i = 1,14 ,
                     4    4          4


      )     590 = 34.7 + 23 6               e#&ន ច/នបផ              7 ចននកង 14 ចនន x1 , x2 , ..., x14 ផ7Gង

      H " | xi |= 3 , )
       7                        590 < 34.8 , មន          ម O ផ5

      .*ច ន | xi | ≤ 3 ច Z គប" i = 1,15

          យ/ង;ន 1215 = x1 + x2 + ... + x15 ≤ 15.34 = 1215 , .* ច 6 | xi | = 3 គប" i = 1,15
                        4    4          4


      .*ច ន        ម          ).5 !&ន 215 U               'ចននគ " &ន^ង ( ±3; ± 3; ... ; ±3)

          cos A cos B cos C 15.17 cos A + 8.17 cos B + 8.15cos C
1.             +     +     =
            8    15    17                   2040
                                                  82 + 152 + 17 2 17
                                                ≤                =
                                                       4080        120
      ព '.*ច ន                82 − 2.8.(17 cos B + 15cos C ) + 152 + 17 2 − 2.15.17 cos A

                = [8 − (17 cos B + 15 cos C ) ] + 152 + 17 2 − 2.15.17 cos A − (17 cos B + 15 cos C ) 2
                                                     2




www.keoserey.wordpress.com
                                                                                                                      Page 18
បជុំវ      គណិតវទ សិស ពូែកគណិតវទ         ក់ទី១០


      )          152 + 17 2 − 2.15.17 cos A − (17 cos B + 15cos C )2

                 = 152 sin 2 C + 17 2 sin 2 B − 2.15.17(cos A + cos B cos C )
                 = 152 sin 2 C + 17 2 sin 2 B − 2.15.17 sin C sin B
                 = (15sin C − 17 sin B ) 2 ≥ 0
                   cos A cos B cos C 17
      .*ច ន             +     +     ≤
                     8    15    17    120
                                           15sin C = 17 sin C
              មiព ក/ &ន5 =)                
                                           8 = 17 cos B + 15cos C
                  15       17
                  sin B = sin C
                                                15    17     8
      ⇔                                      ⇔      =     =
                 8 = 15sin C cos B + 15cos C   sin B sin C sin A
                 
                       sin B
                 15 17 8
          M         =  =
                  b   c a
      .*ច 6                : ABC &ន^ង.* ចនD ង              :).5&ន ជmងSងប             /n 8,15,17

              យ 82 + 152 = 17 2 , 6 !             : ABC )កង           ង" C

           1 1 1  1    1    1 
E.    T =  + +       +    +      = T1 + T2 + T3
           a b c 1+ a 1+ b 1+ c 
                            1         1        1
      ).5 T1 =                    +        +
                        a (1 + a ) b(1 + b) c(1 + c )
                           1         1         1                1          1        1
                 T2 =            +        +          នង T3 =          +         +
                        b(1 + a ) c(1 + b) a (1 + c)         c(1 + a ) a (1 + b) b(1 + c)
              យ 6ទ a, b, c .* ចA 6 យ/ងbចzប&J a ≤ b ≤ c , ព5 6

                 1 1 1     1    1    1
                  ≥ ≥ នង     ≥    ≥
                 a b c   a +1 b +1 c +1
          យ/ង;ន T1 ≥ T2               ព '.* ច ន

                       1         1        1      1             1         1 
      T1 − T2 =              +        +        −          +         +           
                   a (1 + a ) b(1 + b) c(1 + c)  b(1 + a ) c (1 + b) a (1 + c ) 

                   1    1 1  1       1 1  1       1 1
                 =    −      +     −      +     −     
                  1+ a 1+ c  a 1+ b 1+ a  b 1+ c 1+ b  c


www.keoserey.wordpress.com
                                                                                                  Page 19
បជុំវ     គណិតវទ សិស ពូែកគណិតវទ          ក់ទី១០


                1       1  1 1   1         1  1 1 
              =      −       −  +       −       −  ≥ 0
                1 + a 1 + c  a b   1 + b 1 + c  b c 
                         3
       យ/ង;ន T2 ≥                 ព '.* ច ន
                         2
                                         y     z     x
        Z abc = 1 6 =ង a =                 ,b = ,c =          ព5 6
                                         x     y     z
                      x   y   z                   1    1   1       3
              T2 =      +   +     = ( x + y + z)     +   +    −3 ≥
                     y+z z+x x+ y                 y+z z+x x+ y     2
                        3
      .*ចA). T3 ≥
                        2
                      9
      .*ច ន    T≥       ,       មiព ក/ &ន5 =)              a = b = c =1
                      2
K.    =ង      A1 = {k ∈ T / k ⋮ 2} ; A2 = {k ∈ T / k ⋮ 3} ; A3 = {k ∈ T / k ⋮ 5} ;

              A4 = {k ∈ T / k ⋮ 7} ; A5 = {k ∈ T / k ⋮11} ;

       ព5 6          A1 ∪ A2 ∪ A3 ∪ A4 ∪ A5 '        នប,ចនន
                                                        -               4   2កង T )ចក     ច"នDងចនន

      មយកងប,ចនន 2,3,5, 7,11
            -

                                  2010                  2010                 2010
       យ/ង;ន           | A1 | =        = 1005; | A2 | =      = 670; | A3 | =      = 402
                                   2                     3                    5
                                 2010                   2010                  2010 
                       | A4 | =        = 287; | A5 | =  11  = 182; A1 ∩ A2 =  6  = 335;
                                 7                                                 
                         2010                    2010                  2010 
              A1 ∩ A3 =        = 201; A1 ∩ A4 =  14  = 143; A1 ∩ A5 =  22  = 91;
                         10                                                 
                         2010                    2010 
              A2 ∩ A3 =        = 134; A2 ∩ A4 =  21  = 95;
                         15                           
                         2010                   2010 
              A2 ∩ A5 =        = 60; A3 ∩ A4 =  35  = 57;
                         33                          
                         2010                   2010 
              A3 ∩ A5 =        = 36; A4 ∩ A5 =  35  = 57;
                         33                          
                              2010                        2010 
              A1 ∩ A2 ∩ A3 =        = 67; A1 ∩ A2 ∩ A4 =  42  = 47;
                              30                               
                              2010                        2010 
              A1 ∩ A2 ∩ A5 =        = 30; A1 ∩ A3 ∩ A4 =  70  = 28;
                              66                               

www.keoserey.wordpress.com
                                                                                               Page 20
បជុំវ         គណិតវទ សិស ពូែកគណិតវទ                    ក់ទី១០


                                2010                        2010 
                A1 ∩ A3 ∩ A5 =        = 18; A1 ∩ A4 ∩ A5 =  154  = 13;
                                110                              
                                2010                        2010 
                A2 ∩ A3 ∩ A4 =        = 19; A2 ∩ A3 ∩ A5 =  165  = 12;
                                105                              
                                2010                       2010 
                A2 ∩ A4 ∩ A5 =        = 8; A3 ∩ A4 ∩ A5 =  385  = 5;
                                231                             
                                     2010                            2010 
                A1 ∩ A2 ∩ A3 ∩ A4 =        = 9; A1 ∩ A2 ∩ A3 ∩ A5 =  330  = 6;
                                     210                                  
                                     2010                            2010 
                A1 ∩ A2 ∩ A4 ∩ A5 =        = 4; A1 ∩ A3 ∩ A4 ∩ A5 =  770  = 2;
                                     462                                  
                                     2010                                 2010 
                A2 ∩ A3 ∩ A4 ∩ A5 =         = 1; A1 ∩ A2 ∩ A3 ∩ A4 ∩ A5 =         =0
                                     1155 
                                                                           2310 
                                                                                  
                                                       5
      )        យ A1 ∪ A2 ∪ A3 ∪ A4 ∪ A5 =             ∑ Ai    −        ∑           Ai ∩ A j ∩ Ak −
                                                      i =1        1≤i < j < k ≤5

                       −       ∑              Ai ∩ A j ∩ Ak ∩ Aq + A1 ∩ A2 ∩ A3 ∩ A4 ∩ A5 = 1593
                         1≤i < j < k < q ≤5

      .*ច ន         ន T &ន 2010 − 1593 = 417 ចនន).5មន)ចក                                    ច"នDង 2, 3, 5, 7,11

N.        ជ/    /     យ89 ង, ! A ≡ O; B ∈ Oy , D ∈ Ox

                                                              b
          ព5 6       A(0;0), B(0; b), D(d ;0), C (d , b) , M  0;  , N (t ; t + b − d ), P (t ; b)
                                                              2
                MN ⊥ DP ⇔ MN .DP = 0

                               b                                b2
      ⇔        t (t − d ) + b  t + − d  = 0 ⇔ t 2 + (b − d )t +    − bd = 0
                               2                                 2
                AN 2 = t 2 + (t + b − d ) 2 , AD 2 = d 2

          យ/ង;ន AN = AD ⇔                     AN 2 = AD 2 ⇔ d 2 = t 2 + (t + b − d ) 2

                               b2
      ⇔ t 2 + (b − d )t +         − bd = 0 (បIg              e#;ន         យបIក")
                                                                             %
                                2
                                                    '()&




www.keoserey.wordpress.com
                                                                                                                  Page 21
បជុំវ       គណិតវទ សិស ពូែកគណិតវទ           ក់ទី១០



                   វ1       េសរ បឡងអូ            ពិចេវ ត
                                                 ំ                ម*ក់ទី១០ េលកទី XVII

                                                   វ        ទី៤
                                                                   x + y + z = 1 (1)
                                                                  
 .     កU     'ចននព           x; y; z ប      " បព<នQ    ម          x            y       z     9
                                                                   x + yz + y + zx + z + xy = 4 (2)
                                                                  
 .     ក គប"ចននគ "#ជ%&ន p , q , n ច Z                  p, q 'ចននប[ម ./ម0 !

                                         p ( p + 3) + q (q + 3) = n(n + 3)
1.    កងប3ង" គ !ចនច P 2នD ង                 ពន !            :)កង        ម;       ABC (&ន ABC = 900 ;

      AB < 5 ) ផ7GងH "5កq: PA = 2 នង PB = 3
                    7     r

       ក C5ធបផ          ប      "Fង` " PC

E.     គ !ប,ចននព
            -                       a; b; c ផ7GងH "
                                                 7

                       0 < a ≤ b ≤ c; c ≥ 9; 8c ≥ 36 + bc;12c ≥ 36 + bc + 4ac

           យបIក"J
              %               a + b− c ≤0 ,             មiព ក/ &ន 2 ព5,?

K.    កង     នប,ចននគ " ធមn' ព 2 .5" 2011 , គ ជ/
                -                                                            យក 1006 ចនន O/យប ង`/

      '     ន ង A = {a1 ; a2 ; ... ; a1006 }

           យបIក"J កង A &នយក;ន 2 ចនន ./ ម0 !ចននមយ ន )ចក
              %                                                                            ច" នDងចនន

      មយ ទ@

N.     គ ! 6 ចននព           a; b; c; d ; e; f ) ប ប|5 ផ7GងH "5កq:
                                                           7     r

                    a 2 + b 2 − 2a = c 2 + d 2 − 2c = 3; e 2 + f 2 − 10e + 6 f + 33 = 0

          a 2 + c 2 + b 2 + d 2 − 2ac − 2bd = 12; នង (e − 5)(a + c − 2e) + ( f + 3)(b + d − 2 f ) = 0
                                                                    2                 2
                                                   a+c     b+d   
      ក: " C5 *ចបផ                  ប   "ក នyម S =     − e +   −f
                                                    2       2    
                                                 '()&




www.keoserey.wordpress.com
                                                                                                       Page 22
បជុំវ   គណិតវទ សិស ពូែកគណិតវទ                   ក់ទី១០



                                                ចំេលយ

 .       យ x; y; z #ជ%&ន 6

                                    xy   xz       yz      yx         zx     zy
               x + y + z =1 ⇔               +                +                 = 1 (*)
                                     z    y       x       z           y      x

                A      yz      B     zx      C           xy
      =ង tan      =       ; tan =       ; tan =             ).5 0 < A; B; C < π (**)
                2      x       2      y      2           2
                                                     A    B     B   C     C   A
        ម       (1) bច            k/ង#ញ' tan           tan + tan tan + tan tan = 1 (3)
                                                     2    2     2   2     2   2
                                                     1                 1                    1              9
       O/យ      ម     (2) bច             '                    +                    +                   =     (4)
                                                          A                   B                    C       4
                                              1 + tan 2           1 + tan 2            1 + tan 2
                                                          2                   2                    2
                      A    B     C          B    C      A      B C
      (3) ⇔ tan         tan + tan  = 1 − tan tan   ⇔ tan = cot  + 
                      2    2     2          2    2      2      2 2
            ⇔ A + B + C = π + k 2π , =ម (**) 6 A + B + C = π

       ព5 6

                       A        B       C 9   3 + cos A + cos B + cos C 9
      (4) ⇔ cos 2        + cos 2 + cos 2 =  ⇔                          =
                       2        2       2 4               2              4
                                             3             A      A    B−C 3
            ⇔ cos A + cos B + cos C =          ⇔ 1 − 2sin 2 + 2sin cos    =
                                             2             2      2     2   2
                       A         A   B−C
               ⇔ 4sin 2   − 4sin cos     +1 = 0
                       2         2    2
                      A        B−C
                 2sin 2 = cos 2
                                                π
               ⇔                    ⇔ A= B =C =
                 sin B − C = 0                  3
                 
                       2
                                                 1
       បព<នQ    ម     &នច 5/យ x = y = z =
                                                 3
 .    ច Z m 'ចននគ "#ជ%&ន, យ/ង ក;ន                         :5" ព5)ចក m( m + 3) នDង 3

      ពន ! 3 ក :

               m = 3k 6 m(m + 3) ≡ 0 (mod 3)


www.keoserey.wordpress.com
                                                                                                                   Page 23
បជុំវ     គណិតវទ សិស ពូែកគណិតវទ         ក់ទី១០


              m = 3k + 1 6 m(m + 3) ≡ 1 (mod 3)

              m = 3k + 2 6 m(m + 3) ≡ 1 (mod 3)

      .*ច ន    ព5 m )ចក         ច" នDង 3 6 m(m + 3) )ចក         ច" នDង 3 , ព5 m )ចកមន    ច"

      នDង 3 6 m(m + 3) )ចកនDង 3 ;ន            :5"        /n 1

        kប"មក5l " យ/ង#ញ, យ/ង&ន p( p + 3) + q (q + 3) ≡ n(n + 3) (mod 3)

      + ក : : ប/ p នង q           ទQ) )ចកមន        ច" នDង 3 6

              p( p + 3) + q(q + 3) ≡ 2 (mod 3) .

      កង ព5 6 n(n + 3) ≡ 0 (mod 3) M n(n + 3) ≡ 1 (mod 3)

      .*ច ន ក : ន មន ផ7GងH "
                          7

      + ក : : កងព ចនន p, q &នមយ)ចក                   ច" នDង 3

      zប&J p )ចក         ច" នDង 3 , )   p 'ចននប[ម 6             p=3

       ព5 6 18 + q (q + 3) = n(n + 3) , Sញ;ន 18 = n(n + 3) − q(q + 3) (*)

         យ n > q 6 n(n + 3) ≥ q(q + 1)(q + 4) (**)

      ព (*) & (**) យ/ង;ន 18 ≥ 2q + 4 ⇒ q ≤ 7

      .*ច ន    q ∈{2; 3; 5; 7} ( Z q 'ចននប[ម)

      ច Z q = 2 6 18 + 10 = n(n + 3) ⇒ n = 4

      ច Z q = 3 6 18 + 18 = n(n + 3) ⇒ មន&ន n

      ច Z q = 5 6 18 + 40 = n(n + 3) ⇒ មន&ន n

      ច Z q = 7 6 18 + 70 = n(n + 3) ⇒ n = 8

      ពន !.*ចA).        ព5 q = 3

                                       p =3 p =3 p = 2 p = 7
                                                                
      .*ច ន    យ/ង;នប,U
                      -             គM q = 2 ; q = 7 ;  q = 3 ; q = 3
                                       n = 4 n = 8  n = 4 n = 8
                                                                
1.     យ/ង&ន PC )#ងបផ           ព5 P នង C 2&ង&ក" ធ@បនD ងប6 " AB
                                            q             7

        ង"       :)កង       ម;      APQ ( AP = PQ; Q នង B          4     2   ង) មយ ធ@បនDង AP ) ,



www.keoserey.wordpress.com
                                                                                           Page 24
បជុំវ              គណិតវទ សិស ពូែកគណិតវទ        ក់ទី១០


                                        0
                       45
       ចពន !ចtប" បងB5 QA (ផa                     A; ម ងB5 450 ) : P ֏ P ' នង B ֏ B '

      ពន !ចtប" ប)5ងង V A 2 (ផa                   A , ផ5 ធ@ប k = 2 ) : P ' ֏ Q នង B ' ֏ C

       យ/ង;ន PQ = AP = 2 នង QC = 2 P ' B ' = 2 PB = 3 2

          យ PC ≤ PQ + QC ⇒ PC ≤ 2 + 3 2

      .*ច ន      max PC = 2 + 3 2 ទទ5;ន ព5 P                     4   2 5/ប6 " PQ
                                                                           7

      ⇔ AC = AP 2 + PC 2 = 4 + (2 + 3 2)2 = 26 + 12 2

                                  9
                                  c ≤1
                                  
                                  b 9
E.    =មប^ប"Sញ;ន                   + ≤2         (*)
                                   4 c
                                      b 9
                                  a + 4 + c ≤ 3
                                  
                                      9  b   9
      ប)5ង        a + b + 9 = a +
                             
                                   b
                                   4
                                     +  +
                                         
                                       c  4
                                              +  +
                                                 
                                                c
                                                                                  (   c −2   )   9
                                                                                                 c
                             
      Fន# -នj#    មiព Bunhiakovski យ/ង;ន

                 b   9     b 9                            b   9        b 9
                   +   ≤ 2  +  នង                  a+       +   ≤ 3 a + + 
                 4   c     4 3                            4   c        4 3
        ប/ ;     " (*) Sញ;ន                 a + b + 9 ≤ 3+ 2+ c − 2 = 3+ c

      ⇒        a + b− c ≤0 ព ,                   មiព ក/ &ន ព5 a = 1; b = 4; c = 9

K.    + ប)5ង a1 = m1b1; a2 = m2b2 ; ... ; ai = m1006b1006 , កង 6 mi ; bi 'ចននគ "#ជ%&ន,

      bi 'ចនន          &ន C5ព 2 P.5" 2011 , (ច Z i = 1; 2; ...; 1006 )

      + ព 2 P.5" 2011 &ន 1005 ចនន                          ).5   e# ជ/     យក 1006 ចនន               bi

      + =ម ទD     -បទ Dirichlet 6 កងប,ចនន bi ).5 ទ/ប ជ/
                                      -                                                យក&នព ចននគM

      b j = bk ⇒ a j = 2m j b j ; ak = mk bk         (*)

      + zប&J a j > ak            (**)

      + ព (*) នង (**) យ/ង;ន a j )ចក                  ច" នDង ak



www.keoserey.wordpress.com
                                                                                                          Page 25
បជុំវ        គណិតវទ សិស ពូែកគណិតវទ                      ក់ទី១០


N.    កងប3ង"ក: "         យ បព<នQក*F                 ន.              Oxy , ពន ! ងBង" (C1 ) ផa          I1 (1;0) នង

          R1 = 2 , O/យ ងBង" (C2 ) ផa              I 2 (5; −3)       R2 = 1

      ពប^ប" យ/ង;ន A(a; b), B(c; d )                  4    2 5/ (C1 ) នង D(e; f )              4   2 5/ (C2 )

       O/យ AB = 2 3

                                                                             a+c b+d 
       d E 'ចនចក,5 ប
                 -                          "Fង` " AB យ/ង;ន E                   ;    
                                                                              2   2 
                                                                             2
      ពប^ប" យ/ង;ន I 2 D.2 DE = 0 , ព5 6 S = ED                                   = EI 2 − R2 = EI 2 − 1
                                                                                      2    2      2


                                                                                    2
                                                               2  AB 
      =ង (C3 ) ' ងBង"ផa          I1 (1;0)         R3 = I1 E = R1 −     = 4 − 3 =1
                                                                    2 
       យ/ង;ន E         " 2 5/ ងBង" (C3 )

      =ង H ; K 'ប,ចនច ប
                  -                          ពB ប        "ប6 " I1I 2 នDង (C3 ) ( HI 2 > KI 2 )
                                                            7

       យ/ង;ន S ≥ KI 2 − 1 = ( I1 I 2 − R3 ) 2 − 1 = (5 − 1) 2 − 1 = 15
                    2


      min S = 15 ទទ5;ន ព5 E ≡ K O/យ D 'ចនច ប                                        ពB ប    " (C2 ) នDង ងBង" (C ')

      &នFង` "ផa       KI 2

         យ     ម     ប6 " I1I 2 គM 3x + 4 y − 3 = 0 O/យ ងBង" (C3 ) : ( x − 1) 2 + y 2 = 1
                       7

          9 3    a+c 9 b+d    3
      ⇒ K  ;−  ⇒    = ;    =−
          5 5     2  5 2      5
            9+3 3      −3 + 4 3     9−3 3      −3 − 4 3
        a =       ;b =          ;c=       ;d =
               5            5          5            5
      ⇒ 
            9+3 3      −3 + 4 3     9−3 3      −3 − 4 3
        c =       ;d =          ;a=       ;b =
              5            5          5            5
                                                                                                  2           2
                                                                               17      9
         យ     ម      ងBង" (C2 ) : ( x − 5) + ( y + 3) = 1 នង ងBង" (C ') :  x −  +  y +  = 4
                                              2                 2
                                                                                5      5
          24 9 5 3 171      24 9 5    3 171 
      ⇒ D +     ; −
          5 20 3 5 60  M D −
                              5 20 3 ;−  −    
                                      5 60 




www.keoserey.wordpress.com
                                                                                                                    Page 26
បជុំវ    គណិតវទ សិស ពូែកគណិតវទ                 ក់ទី១០


            24 9         5      3 171
        e =   +            ;f =  −
             5 20         3      5 60
      ⇒ 
            24 9         5        3 171
        e =   −            ; f =−  −
            5 20         3        5 60
                                             '()&


                វ1       េសរ បឡងអូ           ពិចេវ ត
                                             ំ                  ម*ក់ទី១០ េលកទី XVII

                                              វ           ទី៥
                                       (3 x + y )( x + 3 y ) xy = 14
                                       
 .             យ បព<នQ    ម            
                                       ( x + y )( x + 14 xy + y ) = 36
                                                    2            2
                                       
 .     ក គប"ប,ចននប[ម p ./ ម0 ! 211 p − 2 )ចក
              -                                                   ច"នDង 11p

1.     គ !      : ABC &នប, ម.uន AA , BB1 , CC1 ប
                          -       1                                           ពBA   ង" ចនច G ( A1 , B1 , C1

        4    2 5/ប, ជmង ប
                   -               "       : ABC )          ប6យ AA , BB1 , CC1
                                                              3   1                       " ងBង" Dក ]

             : ABC =ម5         ប"       ង" A2 , B2 , C2

                         GA2 GB2 GC2
            យបIក"J
               %            +   +    ≥3
                         GA GB    GC
                                                                                                 2
                                      (a + b + c)3  ab + bc + ca 
E.     គ ! a, b, c 'ប,ចននព #ជ%&ន បfញJ
                      -            g              + 2             ≥ 28
                                          abc       a + b2 + c2 
K.     គ ! 1007 ចនច ផpងA 2 5/ប3ង"                      យបIក" J &ន89 ង ច 2011 ចនចក,5
                                                          %                       -

       ផpងAពប,គ* ចនច ន ,
              -                         / ព5,).5&ន 2011 ចនចក,5?
                                                             -

N.    ប6 "ព កង នងព
        7                     ]ប       "ម C ប      "       : ABC              "ប6 " AB
                                                                                 7        ង" E នង D

            យបភ3J ប/ CE = CD 6             AC 2 + BC 2 = 4 R 2 ( R '           ងBង" ក ]
                                                                                     D            :

      ABC )
                                             '()&




www.keoserey.wordpress.com
                                                                                                     Page 27
បជុំវ        គណិតវទ សិស ពូែកគណិតវទ           ក់ទី១០



                                                      ចំេលយ
                                           (3 x + y )( x + 3 y ) xy = 14 (1)
                                           
 .                យ បព<នQ        ម         
                                           ( x + y )( x + 14 xy + y ) = 36 (2)
                                                        2            2
                                           
      =ង        x = u ≥ 0; y = v ≥ 0 , យ/ង;ន

                             (
                 uv 3u 4 + 10u 2 v 2 + 3v 4 = 14
                 
      (1), (2) ⇔ 
                                                      )
                 u 6 + 15u 4 v 2 + 15u 2 v 4 + v 6 = 36
                 
                  3u 5v + 10u 3v3 + 3uv5 = 14
                  
                ⇔ 
                  u + 15u v + 15u v + v = 36
                    6       4 2        2 4   6
                  
                  36 + 2.14 = u 6 + 6u 5v + 15u 4v 2 + 20u 3v3 + 15u 2v 4 + 6uv5 + v 6
                  
                ⇔ 
                  36 − 2.14 + u − 6u v + 15u v − 20u v + 15u v − 6uv + v
                                6      5        4 2        3 3       2 4        5    6
                  
                  
                  (u + v) = 64 = 2
                          6         6
                                                       u + v = 2   u + v = 2
                                                                    
                ⇔                                   ⇔           ∨ 
                                          ( )
                                                 6
                  (u − v) = 8 = 2
                          6
                                                       u − v = 2   u − v = − 2
                                                                    
                  
            យ (u , v ≥ 0 )

                                     2                     2
                    u = 1 +                     u = 1 −
                                    2                     2
                  ⇔                         M   
                    v = 1 −          2          v = 1 +    2
                    
                                    2           
                                                           2
                                            3    3        3    3    
      .*ច ន      បព<នQ&នច 5/យ                + 2; − 2  នង  − 2; + 2 
                                            2    2        2    2    
 .    zប&J p 'ចនន).5                      e# ក, យ/ង;ន            211 p ≡ 2 (mod p ) (1)

      Fន# -នj ទD      បទ Fermat ច Z ចននប[ម p , យ/ង;ន
                      -                                                            211 p ≡ 211 (mod p )   (2)

      ព (1) & (2) Sញ;ន p ' )ចក ប                      " 211 − 2 = 2046 = 2.3.11.31

      )     p 'ចននប[ម 6               p ∈ {2, 3,11,31}

      + ពន ! p = 2

          យ/ង&ន 222 ≡ 2 (mod 22) ,Sញ;ន 221 ≡ 1 (mod 11) (3)

      មu9 ង ទ@       210 ≡ 1 (mod 11) (4)

www.keoserey.wordpress.com
                                                                                                           Page 28
បជុំវ       គណិតវទ សិស ពូែកគណិតវទ         ក់ទី១០



                   ( )
                          2
       6 220 = 210            ≡ 1 (mod 11) , Sញ;ន 221 ≡ 2 (mod 11) , ផ7យនDង (3)

      .*ច ន    p=2        e# 5

      + ពន ! p = 3

                                  ( ) .22 ≡ 4 (mod 11) , .*ច 6
                                        3
      ព (4) យ/ង;ន 232 = 220                                            232 ≡ 1 (mod 11)
                                                                           /

      Sញ;ន 232 ≡ 1 (mod 33) នង 232 ≡ 2 (mod 33)
               /                   /
      .*ច ន      p=3      e# 5

      + ពន ! p = 11

                                              
                                                   ( )
                                           4
       យ/ង&ន 211 p − 2 = 2121 − 2 = 2  230 − 1
                                                           
                               = 2((230 )2 + 1)(230 + 1)(25 + 1)(210 − 25 + 1)(25 − 1)(210 + 25 + 1) .

      ជន      25    យ 32 O/យគបផpនD ង (4) , Sញ;នកងផ5គ: ន &ន) ក=គ:
                                                              -

      25 + 1 ទ).5)ចកនDង 11 ;ន 3 , •ប,ក=គ: ផpង ទ@
                                     - -                                       ទQ) )ចកមន        ច" នDង 11 ,

       6 ផ5គ:)ចកមន                  ច" នDង 11 p = 121

      .*ច ន      p = 11       e# 5

      + ពន ! p = 31

       យ/ង    e#ពន ! ម/5 2340 ≡ 1 (mod 341) M ទ
                                      34
      ព (4) Sញ;ន 2340 = 210                 ≡ 1 (mod 11) (5)

      មu9 ង ទ@     25 ≡ 1 (mod 31) 6 2340 = (25 )68 ≡ 1 (mod 31)             គបផpចនច ន នD ង (5)

      Sញ;នបIg             e#ពន !គMព          ( 11 នង 31     ទQ) 'ប,ចននប[ម)
                                                                   -

      .*ច ន ចននប[ម).5                 e# កគM p = 31

                                             a2                      a2
1.     យ/ង&ន        AA1. A1 A2 = A1 B. A1C =              ⇒ A1 A2 =
                                              4                     4ma

                                        1      a2   a 3   GA2 a 3
      មu9 ង ទ@       GA2 = GA1 + A1 A2 = ma +     ≥     6    ≥
                                        3     4ma    3    GA 2ma



www.keoserey.wordpress.com
                                                                                                    Page 29
Vnmo 30 4-2011-grade 10
Vnmo 30 4-2011-grade 10
Vnmo 30 4-2011-grade 10
Vnmo 30 4-2011-grade 10
Vnmo 30 4-2011-grade 10
Vnmo 30 4-2011-grade 10
Vnmo 30 4-2011-grade 10
Vnmo 30 4-2011-grade 10
Vnmo 30 4-2011-grade 10
Vnmo 30 4-2011-grade 10
Vnmo 30 4-2011-grade 10
Vnmo 30 4-2011-grade 10
Vnmo 30 4-2011-grade 10
Vnmo 30 4-2011-grade 10
Vnmo 30 4-2011-grade 10
Vnmo 30 4-2011-grade 10
Vnmo 30 4-2011-grade 10
Vnmo 30 4-2011-grade 10
Vnmo 30 4-2011-grade 10
Vnmo 30 4-2011-grade 10
Vnmo 30 4-2011-grade 10
Vnmo 30 4-2011-grade 10
Vnmo 30 4-2011-grade 10
Vnmo 30 4-2011-grade 10
Vnmo 30 4-2011-grade 10
Vnmo 30 4-2011-grade 10
Vnmo 30 4-2011-grade 10
Vnmo 30 4-2011-grade 10
Vnmo 30 4-2011-grade 10
Vnmo 30 4-2011-grade 10
Vnmo 30 4-2011-grade 10
Vnmo 30 4-2011-grade 10
Vnmo 30 4-2011-grade 10
Vnmo 30 4-2011-grade 10
Vnmo 30 4-2011-grade 10
Vnmo 30 4-2011-grade 10
Vnmo 30 4-2011-grade 10
Vnmo 30 4-2011-grade 10
Vnmo 30 4-2011-grade 10
Vnmo 30 4-2011-grade 10
Vnmo 30 4-2011-grade 10
Vnmo 30 4-2011-grade 10
Vnmo 30 4-2011-grade 10
Vnmo 30 4-2011-grade 10
Vnmo 30 4-2011-grade 10
Vnmo 30 4-2011-grade 10
Vnmo 30 4-2011-grade 10
Vnmo 30 4-2011-grade 10
Vnmo 30 4-2011-grade 10
Vnmo 30 4-2011-grade 10
Vnmo 30 4-2011-grade 10
Vnmo 30 4-2011-grade 10
Vnmo 30 4-2011-grade 10
Vnmo 30 4-2011-grade 10
Vnmo 30 4-2011-grade 10
Vnmo 30 4-2011-grade 10
Vnmo 30 4-2011-grade 10
Vnmo 30 4-2011-grade 10
Vnmo 30 4-2011-grade 10
Vnmo 30 4-2011-grade 10
Vnmo 30 4-2011-grade 10
Vnmo 30 4-2011-grade 10
Vnmo 30 4-2011-grade 10
Vnmo 30 4-2011-grade 10
Vnmo 30 4-2011-grade 10
Vnmo 30 4-2011-grade 10
Vnmo 30 4-2011-grade 10
Vnmo 30 4-2011-grade 10
Vnmo 30 4-2011-grade 10
Vnmo 30 4-2011-grade 10
Vnmo 30 4-2011-grade 10
Vnmo 30 4-2011-grade 10
Vnmo 30 4-2011-grade 10
Vnmo 30 4-2011-grade 10
Vnmo 30 4-2011-grade 10
Vnmo 30 4-2011-grade 10
Vnmo 30 4-2011-grade 10
Vnmo 30 4-2011-grade 10
Vnmo 30 4-2011-grade 10
Vnmo 30 4-2011-grade 10
Vnmo 30 4-2011-grade 10
Vnmo 30 4-2011-grade 10
Vnmo 30 4-2011-grade 10
Vnmo 30 4-2011-grade 10
Vnmo 30 4-2011-grade 10
Vnmo 30 4-2011-grade 10
Vnmo 30 4-2011-grade 10
Vnmo 30 4-2011-grade 10
Vnmo 30 4-2011-grade 10
Vnmo 30 4-2011-grade 10
Vnmo 30 4-2011-grade 10
Vnmo 30 4-2011-grade 10
Vnmo 30 4-2011-grade 10
Vnmo 30 4-2011-grade 10
Vnmo 30 4-2011-grade 10
Vnmo 30 4-2011-grade 10
Vnmo 30 4-2011-grade 10
Vnmo 30 4-2011-grade 10
Vnmo 30 4-2011-grade 10
Vnmo 30 4-2011-grade 10
Vnmo 30 4-2011-grade 10
Vnmo 30 4-2011-grade 10
Vnmo 30 4-2011-grade 10
Vnmo 30 4-2011-grade 10
Vnmo 30 4-2011-grade 10
Vnmo 30 4-2011-grade 10
Vnmo 30 4-2011-grade 10
Vnmo 30 4-2011-grade 10
Vnmo 30 4-2011-grade 10
Vnmo 30 4-2011-grade 10
Vnmo 30 4-2011-grade 10
Vnmo 30 4-2011-grade 10
Vnmo 30 4-2011-grade 10
Vnmo 30 4-2011-grade 10
Vnmo 30 4-2011-grade 10
Vnmo 30 4-2011-grade 10

More Related Content

Featured

AI Trends in Creative Operations 2024 by Artwork Flow.pdf
AI Trends in Creative Operations 2024 by Artwork Flow.pdfAI Trends in Creative Operations 2024 by Artwork Flow.pdf
AI Trends in Creative Operations 2024 by Artwork Flow.pdfmarketingartwork
 
PEPSICO Presentation to CAGNY Conference Feb 2024
PEPSICO Presentation to CAGNY Conference Feb 2024PEPSICO Presentation to CAGNY Conference Feb 2024
PEPSICO Presentation to CAGNY Conference Feb 2024Neil Kimberley
 
Content Methodology: A Best Practices Report (Webinar)
Content Methodology: A Best Practices Report (Webinar)Content Methodology: A Best Practices Report (Webinar)
Content Methodology: A Best Practices Report (Webinar)contently
 
How to Prepare For a Successful Job Search for 2024
How to Prepare For a Successful Job Search for 2024How to Prepare For a Successful Job Search for 2024
How to Prepare For a Successful Job Search for 2024Albert Qian
 
Social Media Marketing Trends 2024 // The Global Indie Insights
Social Media Marketing Trends 2024 // The Global Indie InsightsSocial Media Marketing Trends 2024 // The Global Indie Insights
Social Media Marketing Trends 2024 // The Global Indie InsightsKurio // The Social Media Age(ncy)
 
Trends In Paid Search: Navigating The Digital Landscape In 2024
Trends In Paid Search: Navigating The Digital Landscape In 2024Trends In Paid Search: Navigating The Digital Landscape In 2024
Trends In Paid Search: Navigating The Digital Landscape In 2024Search Engine Journal
 
5 Public speaking tips from TED - Visualized summary
5 Public speaking tips from TED - Visualized summary5 Public speaking tips from TED - Visualized summary
5 Public speaking tips from TED - Visualized summarySpeakerHub
 
ChatGPT and the Future of Work - Clark Boyd
ChatGPT and the Future of Work - Clark Boyd ChatGPT and the Future of Work - Clark Boyd
ChatGPT and the Future of Work - Clark Boyd Clark Boyd
 
Getting into the tech field. what next
Getting into the tech field. what next Getting into the tech field. what next
Getting into the tech field. what next Tessa Mero
 
Google's Just Not That Into You: Understanding Core Updates & Search Intent
Google's Just Not That Into You: Understanding Core Updates & Search IntentGoogle's Just Not That Into You: Understanding Core Updates & Search Intent
Google's Just Not That Into You: Understanding Core Updates & Search IntentLily Ray
 
Time Management & Productivity - Best Practices
Time Management & Productivity -  Best PracticesTime Management & Productivity -  Best Practices
Time Management & Productivity - Best PracticesVit Horky
 
The six step guide to practical project management
The six step guide to practical project managementThe six step guide to practical project management
The six step guide to practical project managementMindGenius
 
Beginners Guide to TikTok for Search - Rachel Pearson - We are Tilt __ Bright...
Beginners Guide to TikTok for Search - Rachel Pearson - We are Tilt __ Bright...Beginners Guide to TikTok for Search - Rachel Pearson - We are Tilt __ Bright...
Beginners Guide to TikTok for Search - Rachel Pearson - We are Tilt __ Bright...RachelPearson36
 
Unlocking the Power of ChatGPT and AI in Testing - A Real-World Look, present...
Unlocking the Power of ChatGPT and AI in Testing - A Real-World Look, present...Unlocking the Power of ChatGPT and AI in Testing - A Real-World Look, present...
Unlocking the Power of ChatGPT and AI in Testing - A Real-World Look, present...Applitools
 
12 Ways to Increase Your Influence at Work
12 Ways to Increase Your Influence at Work12 Ways to Increase Your Influence at Work
12 Ways to Increase Your Influence at WorkGetSmarter
 

Featured (20)

AI Trends in Creative Operations 2024 by Artwork Flow.pdf
AI Trends in Creative Operations 2024 by Artwork Flow.pdfAI Trends in Creative Operations 2024 by Artwork Flow.pdf
AI Trends in Creative Operations 2024 by Artwork Flow.pdf
 
Skeleton Culture Code
Skeleton Culture CodeSkeleton Culture Code
Skeleton Culture Code
 
PEPSICO Presentation to CAGNY Conference Feb 2024
PEPSICO Presentation to CAGNY Conference Feb 2024PEPSICO Presentation to CAGNY Conference Feb 2024
PEPSICO Presentation to CAGNY Conference Feb 2024
 
Content Methodology: A Best Practices Report (Webinar)
Content Methodology: A Best Practices Report (Webinar)Content Methodology: A Best Practices Report (Webinar)
Content Methodology: A Best Practices Report (Webinar)
 
How to Prepare For a Successful Job Search for 2024
How to Prepare For a Successful Job Search for 2024How to Prepare For a Successful Job Search for 2024
How to Prepare For a Successful Job Search for 2024
 
Social Media Marketing Trends 2024 // The Global Indie Insights
Social Media Marketing Trends 2024 // The Global Indie InsightsSocial Media Marketing Trends 2024 // The Global Indie Insights
Social Media Marketing Trends 2024 // The Global Indie Insights
 
Trends In Paid Search: Navigating The Digital Landscape In 2024
Trends In Paid Search: Navigating The Digital Landscape In 2024Trends In Paid Search: Navigating The Digital Landscape In 2024
Trends In Paid Search: Navigating The Digital Landscape In 2024
 
5 Public speaking tips from TED - Visualized summary
5 Public speaking tips from TED - Visualized summary5 Public speaking tips from TED - Visualized summary
5 Public speaking tips from TED - Visualized summary
 
ChatGPT and the Future of Work - Clark Boyd
ChatGPT and the Future of Work - Clark Boyd ChatGPT and the Future of Work - Clark Boyd
ChatGPT and the Future of Work - Clark Boyd
 
Getting into the tech field. what next
Getting into the tech field. what next Getting into the tech field. what next
Getting into the tech field. what next
 
Google's Just Not That Into You: Understanding Core Updates & Search Intent
Google's Just Not That Into You: Understanding Core Updates & Search IntentGoogle's Just Not That Into You: Understanding Core Updates & Search Intent
Google's Just Not That Into You: Understanding Core Updates & Search Intent
 
How to have difficult conversations
How to have difficult conversations How to have difficult conversations
How to have difficult conversations
 
Introduction to Data Science
Introduction to Data ScienceIntroduction to Data Science
Introduction to Data Science
 
Time Management & Productivity - Best Practices
Time Management & Productivity -  Best PracticesTime Management & Productivity -  Best Practices
Time Management & Productivity - Best Practices
 
The six step guide to practical project management
The six step guide to practical project managementThe six step guide to practical project management
The six step guide to practical project management
 
Beginners Guide to TikTok for Search - Rachel Pearson - We are Tilt __ Bright...
Beginners Guide to TikTok for Search - Rachel Pearson - We are Tilt __ Bright...Beginners Guide to TikTok for Search - Rachel Pearson - We are Tilt __ Bright...
Beginners Guide to TikTok for Search - Rachel Pearson - We are Tilt __ Bright...
 
Unlocking the Power of ChatGPT and AI in Testing - A Real-World Look, present...
Unlocking the Power of ChatGPT and AI in Testing - A Real-World Look, present...Unlocking the Power of ChatGPT and AI in Testing - A Real-World Look, present...
Unlocking the Power of ChatGPT and AI in Testing - A Real-World Look, present...
 
12 Ways to Increase Your Influence at Work
12 Ways to Increase Your Influence at Work12 Ways to Increase Your Influence at Work
12 Ways to Increase Your Influence at Work
 
ChatGPT webinar slides
ChatGPT webinar slidesChatGPT webinar slides
ChatGPT webinar slides
 
More than Just Lines on a Map: Best Practices for U.S Bike Routes
More than Just Lines on a Map: Best Practices for U.S Bike RoutesMore than Just Lines on a Map: Best Practices for U.S Bike Routes
More than Just Lines on a Map: Best Practices for U.S Bike Routes
 

Vnmo 30 4-2011-grade 10

  • 1. បជុំវ គណិតវទ សិស ពូែកគណិតវទ ក់ទី១០ បកែ បេ យ ែកវ សិរ" គណិតវទ អូ ំពិក បៃពណីេវៀត ម 30-4 ំ 2011
  • 2. បជុំវ គណិតវទ សិស ពូែកគណិតវទ ក់ទី១០ វ អូ ពិក បៃពណីេវ ត ំ មេលកទី XVII ំ ២០១១ មុខ វ%ៈ គណិត វទ)*ក់ទី ១០ រយៈេពល ១៨០/ទី & . យ ម ង មកង នចននព x3 − 15 x2 + 78 x − 141 = 53 2 x − 9 . គ !ចននគ "#ជ%&ន n នង d1 < d 2 < d3 < d 4 'បន )ចកគ "#ជ%&ន * ចបផ ប " n ក គប"ប,ចននគ "#ជ%&ន n ./ម0 ! n = d1 + d 2 + d3 + d 4 - 2 2 2 2 1. 2កងប3ង" គ !ម xOy នងព ចនច A 4 2 5/កន3 ប6 " Ox នង B 7 4 2 5/កន3 ប6 " Oy 89 ង, ! 7 : OAB ម; ង" O ∆ 'ប6 "ច5< មយមន 7 " =ម O , ) ?) ង " =មចនចក,5 I ប - " AB នង " កន3 ប6 " Ox, Oy @ងA 7 ង"ប,ចនច C , D - =ង M 'ចនចក,5 ប - " CD, N 'ចន ប ពBCន OM នDង AB, H 'ច ,5)កង ប " N 2 5/ CD ព5 ∆ ច5< , ច* ក នចនច ប " ចនច H E. គ ! a, b, c 'បចននព មនF#ជ%&ន ផ7GងH " a 2 + 4b 2 + 9c 2 = 14 7 យបIក"J % 3b + 8c + abc ≤ 12 K. យបIក"Jព 2011 ចននគ "#ជ%&ន,កL % យ គ) ង ជ/ / ;នចននព ).5ផ5ប* ក Mផ5.ក ប "?)ចក ច" នDង 4018 x2 y 2 N. គ ! F5ប ( E ) : + = 1 នងប6 " ( ∆ ) : 2 x − 2 y + 4 = 0 7 =ង B, C @ង 8 4 A'ចនច ប ពB ប " ( ∆ ) នង ( E ) , yB > yC O/យ A 'ចនច 2 5/ ( E ) 89 ង , ! ប)#ងព A P ( ∆ ) )#ងបផ កចនច M 2 5/ ( E ) ./ ម0 ! ប)#ងព M Pប6 " AB គM)#ងបផ 7 '()& www.keoserey.wordpress.com Page 1
  • 3. បជុំវ គណិតវទ សិស ពូែកគណិតវទ ក់ទី១០ ចំេលយ . ម ).5 ! មម* 5នD ង ( ( x − 5)3 = −3 x + 5 5 + 3 2 x − 9 − 9 ) =ង y = 5 + 3 2 x − 9 , យ/ង;ន បព<នQ ម ( x − 5)3 = −3x + 5 y − 9   ( y − 5) = 2 x − 9 3  .កFងRនDងFងR ម (1) នង (2) , យ/ង;ន ( x − 5)3 − ( y − 5)3 = −5 x + 5 y . ⇔ ( x − y )  ( x − 5)2 + ( x − 5)( y − 5) + ( y − 5)2 + 5 = 0   យ ( x − 5) 2 + ( x − 5)( y − 5) + ( y − 5)2 + 5 = 2  1  3 =  x − 5 + ( y − 5)  + ( y − 5) 2 + 5 > 0, ∀x, y ∈ ℝ  2  4 6 ម (3) មម*5នDង x = y ព (2) Sញ;ន ( x − 5)3 = 2 x − 9 ⇔ x3 − 15 x 2 + 73x − 116 = 0 x = 4 ⇔ ( x − 4)( x − 11x + 29) = 0 ⇔  2  x = 11 ± 5   2  11 − 5 11 + 5    .*ច ន នU ប " ម ).5 !គM S = 4; ;    2 2   . ឃ/ញJ x 2 ≡ 0 (mod 4) ព5 x គ*, x 2 ≡ 1 (mod 4) ព5 x ប/ n 'ចនន 6 គប" ប,ចនន di - ទQ) O/យ n ≡ d12 + d 22 + d 32 + d 4 ≡ 1 + 1 + 1 + 1 ≡ 0 (mod 4) (ក : ន ផ7យព 2 ព ) .*ច ន យ/ង;ន n = 2k ប/ 4 ' )ចក ប " n 6 d1 = 1 នង d 2 = 2, n ≡ 1 + 0 + d 32 + d 42 )ចកមន ច" នDង 4 (ក : ន ផ7យព ព ) .* ច ន យ/ង;ន n )ចកមន ច" នDង 4 .*ច ន {d1 , d 2 , d3 , d 4 } = {1, 2, p, q} www.keoserey.wordpress.com Page 2
  • 4. បជុំវ គណិតវទ សិស ពូែកគណិតវទ ក់ទី១០ M {d1 , d 2 , d 3 , d 4 } = {1, 2, p, 2 p} ចZ p, q 'ប,ចននប[ម - កងក : {d1 , d 2 , d 3 , d 4 } = {1, 2, p, q} យ/ង;ន n ≡ 3 (mod 4) (ផ7យព ព ) .*ច ន n = 5 (1 + p 2 ) O/យ n )ចក ច" នDង 5 , 6 p = d3 = 5 នង n = 130 1. • យបIក")ផក % ប =ង O + P 'ចនច ប ពB ប "កន3 ប6 " OI 7 C1 នDង ងBង" (C ) Dក ] : OCD A D1 E N + J 'ចនច ប ពB ប "កន3 ប6 " OI 7 I D H នDងប6 ")កងនD ងកន3 ប6 " Ox 7 7 ង" A H1 B F + E , F @ងA'ច ,5)កង ប " P H2 x P P 5/កន3 ប6 " Ox, Oy 7 y =មប^ប" យ/ង;ន ប/ C ≡ A , 6 ! D ≡ B ( ផ7យមក#ញ) 6 M ≡ N ≡ H ≡ I M ប/ C ≠ A នង D ≠ B , ព5 6 , + កន3 ប6 " OI 'ប6 "ព ប 7 7 "ម AOB ⇒ P 'ចនចក,5ធ* CD ប - " (C ) ⇒ PM ⊥ CD ព 6 , E, M , F 4 2 5/ប6 " Simson ប 7 "ចនច P ច Z ∆OCD + កL Z កន3 ប6 " OI 'ប6 " ព ប 7 7 " AOB 6 EF ⊥ OI ⇒ EF || AB + AJ , EP )កង មA PនDង Ox 6 AJ || EP ព 6 =ម ទD -បទ= 5 យ/ង;ន OJ OA ON = = ⇒ NJ || PM ⇒ NJ ⊥ CD ⇒ N , H , J " ង"ជ OP OE OM .*ច ន H 4 2 5/ ងBង" (T ) Fង` " ផa IJ យប, កន3 ប6 " Ox, OI នងប, - 7 - ចនច A, B 2នDង 6 I , J 2នDង ⇒ (T ) 2នD ង • 5ម ប " នចនច =ង C1 , D1 @ងA'ប,ចនច - 4 2 5/ Ox, Oy 89 ង, ! IC1 || Oy នង ID1 || Ox; H1 , H 2 @ងA'ចនច ប ពB ប " (T ) នDង IC1 នង ID1 ព5 6 C bច 4 2Fង` " www.keoserey.wordpress.com Page 3
  • 5. បជុំវ គណិតវទ សិស ពូែកគណិតវទ ក់ទី១០ OC1 ) ប9 , .*ចAនD ង D bច c 4 2 ]Fង` " OD1 ) ប9 ,c Sញ;ន H 4 2 5/ធ* H1 H 2 &នផ7ក I ប " ងBង" (T ) ( 5/ក)5ងព ចនច H1 , H 2 ) • យបIក")ផកផ7យ % 2 5/ធ* H1 H 2 &នផ7ក I ប " ងBង" (T ) យ/ង dចនច H ' =ង C ', D ' 'ចនច ប ពB ប " IH ' @ងA PនD ង Ox, Oy; P ' 'ចនច ប ពB ប "កន3 ប6 " OI PនD ង ងBង" Dក 7 ] ∆OC ' D '; E′; F ′ @ងA'ច ,5)កង ប " P ' P 5/ Ox, Oy; N ′ 'ចនច ប ពB ប " JH ' PនD ង AB; M ′ 'ចនចក,5 ប - " C 'D' យ/ង e# បfញJ O, N ', M ' g " ង"ជ A ព '.*ច ន =ង N '' 'ចនច ប ពB ប " OM ' នង AB ព5 6 =ម ^ប" បIក" % )ផក ប យ/ង;ន OJ ' OA ON '' = = ⇒ N '' J || P ' M ' OP ' OE ' OM ' ⇒ N '' J ⊥ C ' D ' ⇒ N '', H ′, J " ង"ជ ⇒ N '' ≡ N ' .*ច ន O, N ', M ' " ង" ជ A (បIg e# យបIក") % • ន hន នចនច H 'ធ* H1 H 2 &នផ7កចនច I ប " ងBង" (T ) ( 5/ក)5ងព ចនច H1 , H 2 ) E. # មiព).5 ! មម* 5នD ង 6b + 16c + 2abc ≤ 24 (1) + Fន# -នj# មiព Cauchy យ/ង;ន 6b + 16c ≤ 3(b2 + 1) + 8(c 2 + 1) = 11 + 3b 2 + 8c 2 . = 11 + (a 2 + 4b 2 + 9c 2 ) − a 2 − b2 − c 2 = 25 − a 2 − b2 − c 2 . 6 ./ ម0 យបIក" (1) យ/ង Aន") % e# យបIក"# % មiព a 2 + b2 + c 2 − 1 ≥ 2abc (2) , ច Z a, b, c មនF#ជ%&ន + =មប^ប" a 2 + 4b2 + 9c 2 = 14 , # មiព (2) bច k/ង#ញ.* ច ង ម 14(a 2 + b2 + c 2 ) − (a 2 + 4b2 + 9c 2 ) ≥ 28abc . ⇔ 13a 2 + 10b 2 + 5c 2 ≥ 28abc . ⇔ (13a 2 + 10b 2 + 5c 2 ) a 2 + 4b 2 + 9c 2 ≥ 28 14abc www.keoserey.wordpress.com Page 4
  • 6. បជុំវ គណិតវទ សិស ពូែកគណិតវទ ក់ទី១០ + Fន# -នj# មiព Cauchy ម-ង ទ@ ច Z 28 ចនន យ/ង;ន 13a 2 + 10b 2 + 5c 2 ≥ 2828 ( a 2 )13 (b 2 )10 (c 2 )5 = 2814 a13b10 c 5 ( ) 2 នង a 2 + 4b 2 + 9c 2 ≥ 1414 a 2 (b 2 ) 4 (c 2 )9 = 14 14 ab 4 c 9 ( ) 2 + .* ច 6 (13a 2 + 10b 2 + 5c 2 ) a 2 + 4b 2 + 9c 2 ≥ 2814 a13b10 c 5 14 14 ab 4 c 9 = 28 14abc មiព ក/ &ន ព5 (a, b, c) = (1,1,1) 5l " e#;ន យបIក" ច^5" % K. ព5)ចកចននគ "#ជ%&ន,មយ នD ង 4018 6 ប,- :5" e# 4 2កង ន {0, 1, ..., 4017} កងប,- :5" ង 5/ យ/ង)ចក ចញ' កmម.* ច ង ម + កmមទមយ ម&នប,ចនន ព5)ចកនD ង 4018 ;ន - :5" /n 0 + កmមទព ម&នប,ចនន ព5)ចកនD ង 4018 ;ន - :5" /n 1 M /n 4017 + កmមទប ម&នប,ចនន ព5)ចកនD ង 4018 ;ន - :5" /n 2 M /n 4016 ............ + កmមទ 2009 ម&នប,ចនន ព5)ចកនD ង 4018 ;ន - :5" /n 2008 M /n 2010 + កmមទ 2010 ម&នប,ចនន ព5)ចកនD ង 4018 ;ន - :5" /n 2009 .*ច ន &នSងF " 2010 កmម, ) )ប '&ន 2011 ចនន 6 =ម ទD -បទ Dirichlet ?ងពក? e#&នព ចនន ).5 :5"កង ប&:#ធ)ចកនD ង 4018 oក"ច*5កង កmម 3 'មយA ន គM'ព ចនន).5 e# ក Z ប/ ព ចនន ន &ន :5" /nA 6 ផ5 ងប " ពក?)ចក ច"នDង 4018 , ប/ ពក?&ន :5" ផpងA 6 ផ5ប* ក ប "ពក?)ចក ច" នDង 4018 N. =ង A(2 2 sin t ; 2 cos t ) ∈ ( E ), t ∈ [ 0; 2π ]  π 4 2 sin  t −  + 4 4sin t − 4 cos t + 4  4 យ/ង&ន d ( A, (∆)) = = 6 6 www.keoserey.wordpress.com Page 5
  • 7. បជុំវ គណិតវទ សិស ពូែកគណិតវទ ក់ទី១០ .*ច ន d ( A;(∆)) ធបផ ព5 π 3π sin(t − ) = 1 ⇒ t = ⇒ A(2; − 2) 4 4 t = 0 ⇒ B(0; 2)  π 1 d ( A; (∆)) = 0 ⇒ sin  t −  = − ⇒  3π  4 2 t = ⇒ C (−2 2; 0)  2 ម ( ) ( AB ) : 2 + 2 x + 2 y − 4 = 0  π π d ( M , ( AB )) = ( ) 2 + 2 2 2 sin t + 4 cos t − 4 = 8 2 sin  t +  cos − 4  8 8 10 + 4 2 10 + 4 2 .*ច ន d ( M , ( AB)) )#ងបផ ព5 π 11π 3π 3π sin(t + ) = −1 ⇒ t = ⇒ M (−2 2 sin ; − 2 cos ) 8 8 8 8 '()& វ1 េសរ បឡងអូ ពិចេវ ត ំ ម*ក់ទី១០ េលកទី XVII វ ទី១ . យ ម ង ម 5/ នចននព ( 9 x3 + 8 = 2 x 2 + 8 ) (1) . បfញJ មន&នប,ចននគ " x, y, z ផ7GងH " ទ6ក"ទនង g - 7 ( x + 2010)2 + ( x + 2012)2 = ( x + y + z + 2008)( y + z − x − 2014) 1. គ ! : ABC Dកកង ងBង" (O) នង ក ] ងBង" ( I ) D =ង D , E , F @ង A'ប,ចនចប9 ប - " ( I ) PនD ងប, ជmង BC , CA, AB - ង" ងBង" (O1 ) ប9 ]នDង ( I ) ង"ចនច D O/យប9 កងនDង (O) ង"ចនច K , ងBង" (O2 ) ប9 ]នD ង (I ) ង" ចនច E O/យប9 ]នD ង (O) ង" M , ងBង" (O3 ) ប9 ]នD ង ( I ) ង" F O/យប9 កងនD ង (O) ង" ចនច N បfញJ g a ). ប,ប6 " DK , EM , FN - 7 "A ង"ចនច P b). ប6 " OP 7 "=មF * ង" H ប " : DEF E. គ ! a, b, c 'ចននព មនF#ជ%&នប).5 ផ7GងH " a 2 + 4b 2 + 9c 2 = 14 7 www.keoserey.wordpress.com Page 6
  • 8. បជុំវ គណិតវទ សិស ពូែកគណិតវទ ក់ទី១០ បfញJ g 3b + 8c + abc ≤ 12 2011 2 K. គ !Fនគមនj F ( x) = ∑ (k − 2011x) C2011 xk (1 − x)2011−k k k =0 ក C5ធបផ ប "Fនគមនj 2 5/ច 63 [0;1] '()& ចំេលយ . 5កq: x ≥ −2 r ម មម*5នDង 9 ( x + 2)( x 2 − 2 x + 4) = 2  2( x + 2) + x 2 − 2 x + 4 (2)   យ x 2 − 2 x + 4 = ( x − 1) 2 + 3 ≥ 3 )ចកFងRSងព Cន ម (2) នDង x 2 − 2 x + 4 , យ/ង;ន  x+2  x+2 4 2 −9 2 +2=0  x − 2x + 4  x − 2x + 4 x+2 1 ⇒ = ⇒ x = 9 ± 109 (យក) x2 − 2 x + 4 16 . ម ).5 ! មម* 5នD ង ( x + 2010 )2 + ( x + 2012 )2 = ( y + z − 3)2 − ( x + 2011)2 ⇔ ( x + 2010 )2 + ( x + 2011)2 + ( x + 2012 )2 = ( y + z − 3)2 ⇔ 3 x 2 + 12066 x + 20102 + 20112 + 20122 = ( y + z − 3) 2 FងR ង ឆBង ប " ម )ចកនD ង 3 ;ន :5" /n 2 , FងR ង - ប " ម )ចកនDង 3 ;ន :5" /n 1 M /n 0 .*ច ន ម ).5 !AនU n 'ចននគ " 1. a ). .ប*ង យ/ង យបIក" Lemma : " ! X , Y 'ព ចនច 2 5/ ងBង" (O) , ងBង" (O ') % មយ ប9 នDង XY ង" U O/យប9 កងនD ង (O) ង" V ព5 6 , ប6 " UV 7 " =ម ចនចក,5 Z ប - "ធ* XY មន&នផ7ក V " www.keoserey.wordpress.com Page 7
  • 9. បជុំវ គណិតវទ សិស ពូែកគណិតវទ ក់ទី១០ ព '.*ច ន , ពន !ចtប" ប)5ងងផa V : (O ') → (O) ព5 6 XY → d ផ7GងH " d || XY O/យ d ប9 នD ង (O) 7 ង" Z ' បiព ប * " U ⇒ Z 'ចនចក,5 - ប "ធ* XY A A1 V O' B0 B Y C0 M X O3 O O N E O2 I C1 B1 D Z B C O1 K A0 ពន !5l ").5 ! =ង A , B1 , C1 'ចនច ប 1 ពB ប " DK , EM , FN PនD ង (O) , =ម Lemma យ/ង ;ន A , B1 , C1 'ប,ចនចក,5 ប 1 - - "ធ* BAC , CBA, ACB =ង A0 , B0 , C0 'ប,ចនចឆ3 ប - " A , B1 , C1 ធ@បនDង O , ព5 6 ∆A0 B0C0 , ∆A B1C1 &ន 1 1 ប, ជmង - បA មu9 ង ទ@ B0C0 , EF ⊥ AI ; A0C0 , FD ⊥ BI ; A0 B0 , DE ⊥ CI Sញ;ន ∆A0 B0C0 , ∆DEF &នប, ជmង - បA .* ច 6 ∆A B1C1 , ∆DEF 1 &នប, ជmង - បA O/យមន /A ( ∆A B1C1 Dកកង (O), ∆DEF Dកកង ( I ) ) n 1 ⇒ ∃ ចtប"ប)5ងង ប)5ង ∆DEF P' ∆A1B1C1 ⇒ DA1 , EB1 , FC1 ប ពBA ង"ផa P ប "ចtប" ប)5ងង b). ព ន a). ⇒ P, O, I " ង" ជ A (1) =ង A ', B ', C ' 'ប,ចនច ប - ពB ប "ប,កព - " DD ', EE ', FF ' ប " ∆DEF នDង (1) យ EE ' FF ' កកង 6 B ′A′D = B ′ED = C ′FD = C ′A′D ⇒ D ' D ចនចក,5 ប - "ធ* B ' C ' ⇒ B ′C ′ ⊥ ID ⇒ B ′C || BC ( Z BC ⊥ ID ) .*ចA). C ' A ' || CA, A ' B ' || AB , O/យ H 'ផa ងBង" កកង ∆A′B ′C ′ D ព 6 ∆ABC , ∆A′B′C ′ &នប, ជmង - បA នងមន /nA ( Z ∆ABC Dក www.keoserey.wordpress.com Page 8
  • 10. បជុំវ គណិតវទ សិស ពូែកគណិតវទ ក់ទី១០ កង (O), ∆A′B ′C ' Dកកង ( I ) ) ⇒ ∃ ចtប" ប)5ងង ប)5ង ∆ABC P' ∆A′B ′C ′ 6 ! AA ', BB ', CC ' ប ពBA ង" ផa ប)5ងង Q ⇒ Q , I , O " ង" ជ A (2) មu9 ង ទ@ , I , H 'ផa Dកកង ∆ABC , ∆A′B′C ′ ⇒ Q, H , I " ង" ជ A (3) ព (1), (2), (3) យ/ង;នបIg e#;ន យបIក" % E. # មiព).5 ! មម* 5នD ង 6b + 16c + 2abc ≤ 24 (1) + Fន# -នj# មiពក* យ/ង;ន 6b + 16c ≤ 3(b 2 + 1) + 8(c 2 + 1) = 11 + 3b 2 + 8c 2 = 11 + (a 2 + 4b 2 + 9c 2 ) − a 2 − b 2 − c 2 = 25 − a 2 − b 2 − c 2 6 ./ ម0 យបIក" (1) យ/ង Aន") % e# យបIក"# % មiព a 2 + b 2 + c 2 − 1 ≥ 2abc (2) , ច Z a, b, c មនF#ជ%&ន + =មប^ប" a 2 + 4b 2 + 9c 2 = 14 ,# មiព (2) bច k/ង#ញ.* ច ង ម ( ) ( ) 14 a 2 + b 2 + c 2 − a 2 + 4b 2 + 9c 2 ≥ 28abc ⇔ 13a 2 + 10b 2 + 5c 2 ≥ 28abc ⇔ (13a2 + 10b2 + 5c2 ) a 2 + 4b 2 + 9c 2 ≥ 28 14abc + Fន# -នj# មiពក* ម-ង ទ@ ច Z 28 ចនន យ/ង;ន ( ) (b2 ) ( c2 ) 13 20 5 14 13a 2 + 10b 2 + 5c 2 ≥ 2828 a 2 = 28 a13b10c5 ( ) ( c2 ) ( ) 4 9 2 14 នង a 2 + 4b 2 + 9c 2 ≥ 1414 a 2 b 2 = 14 ab 4c9 + .*ច 6 (13a2 + 10b2 + 5c2 ) ( ) 2 a 2 + b 2 + c 2 ≥ 2814 a13b10c5 14 14 ab 4 c 9 = 28 14abc # មiព ក/ &ន ព5 (a, b, c) = (1,1,1) , 5l " e#;ន យបIក" % n K. យ/ង&ន A= ∑ (k − nx)2Cnk xk (1 − x)n−k k =0 www.keoserey.wordpress.com Page 9
  • 11. បជុំវ គណិតវទ សិស ពូែកគណិតវទ ក់ទី១០ n n n = (nx) 2 ∑ Cn x k (1 − x)n − k k +∑k 2 Cn x k (1 − x) n −k k − 2nx ∑ kCn x k (1 − x)n − k k k =0 k =0 k =0 n n ពន ! A1 = ∑ kCnk xk (1 − x)n−k = ∑ kCnk xk (1 − x)n−k k =0 k =1 n n = n∑ Cn −1 x k (1 − x)n −k k −1 = nx ∑ Cn −1 (1 − x)n − k k −1 k =1 k =1 n−k = nx [ x + (1 − x) ] = nx n n A2 = ∑k 2 Cn x k (1 − x)n − k k = ∑ k 2Cn x k (1 − x)n − k k k =0 k =1 n = n ∑ Cn −1 x k (1 − x)n − k k −1 k =1 n n = n ∑ Cn −1 x k (1 − x)n − k + n ∑ (k − 1)Cn −1 x k (1 − x) n − k k −1 k −1 k =1 k =1 n = nx + n ∑ (k − 1)Cn −1 x k (1 − x) n − k k −1 k =1 n = nx + n(n − 1) ∑ Cn − 2 x k (1 − x)n − k = nx + n(n − 1) x 2 k −2 k =2 n ∑ Cnk xk (1 − x)n−k = [ x + (1 − x)] n A3 = =1 k =0 .*ច ន A = (nx )2 + nx + n(n − 1) x 2 − 2(nx )2 = nx(1 − x) Fន# -នj5ទQផ5 ង 5/ យ/ង;ន f ( x ) = 2011x(1 − x) យ x ∈ [ 0;1] 6 x,1 − x ≥ 0 ព 6 =ម# មiពក* 2  x + (1 − x)  2011 1 f ( x) ≤ 2011.   = , Iw មiព ក/ &ន ព5 x =  2  4 2 2011 1 .*ច ន max f ( x) = ទទ5;ន ព5 x = 4 2 '()& www.keoserey.wordpress.com Page 10
  • 12. បជុំវ គណិតវទ សិស ពូែកគណិតវទ ក់ទី១០ វ1 េសរ បឡងអូ ពិចេវ ត ំ ម*ក់ទី១០ េលកទី XVII វ ទី២  x + y + 1 + 1 = 4( x + y )2 + 3. x + y  . យ បព<នQ ម  30 x − 4 y = 2011  . ក គប"ប,ចនន).5&ន 5ខបខ7ង")ចក - ច" នDង 11 89 ង, !ផ5)ចក ប "? /nនDងផ5ប*ក ប "ប, 5ខ=មខ7ង" ប - "? 1. គ ! : ABC ).5 A 'ម * ចបផ , កកង ងBង" (O), ចនច D ច5< D 2 5/ ធ* * ច BC ម.uទ< ប " AB នង AC " AD @ងA ង" E នង F =ង T ' ចនច ប ពB ប " BE នង CF ប6 " 7 "=ម T O/យ បនD ង AB " AD ង" N ង" ប 5k* ម TNDM =ង P 'ចនចក,5 ប - " MC O/យ I 'ចនច ប ពB ប " PT នD ង ម.uទ< ប " OP យបIក"J I ច5< % 2 5/)ខpនD ងមយ 2 1 4 E. គ ! x, y, z 'ប,ចននព #ជ%&ន ផ7GងH " ប,5កq: - 7 - r + ≤ 1 នង + y ≤ 2 , x y z ក C5 *ចបផ ប "ក នyម P ( x, y , z ) = x + 9 y + z K. យបIក"J កង 17 ចននគ " ,កL % យ ) ង&ន 9 ចនន).5&នផ5ប* ក )ចក ច"នDង 9 N. ប,កព*5 ប - " ∆ABC 'ចនច).5&នក* F ន'ចននគ " O/យមន&ន :, * ច'ង ∆ABC នង&ន^ង.*ចនDង ∆ABC ទ ច Z កព*5 ប "' ប,ចនច&នក* F - ន'ចននគ " បfញJ ផa g D ប " ងBង" ក ] ប D " ∆ABC មន)មន'ចនចមយ).5&នក*F ន'ចននគ " ទ '()& www.keoserey.wordpress.com Page 11
  • 13. បជុំវ គណិតវទ សិស ពូែកគណិតវទ ក់ទី១០ ចំេលយ  x + y + 1 + 1 = 4( x + y )2 + 3. x + y  . យ បព<នQ ម  (1) 30 x − 4 y = 2011  =ង u = x+ y ≥0  u 2 + 1 + 1 = 4u 4 + 3u (2)   (1) ⇔ 30u 2 − 34 y = 2011  x + y = u 2  (2) ⇔ u 2 + 1 + 1 = 4u 4 + 3u ⇔ u 2 + 1 − 3u + 1 − 4 y 4 = 0 u 2 + 1 − 3u 2 ⇔ + (1 − 2u 2 )(1 + 2u 2 ) = 0 u + 1 + 3u 2  1  ⇔ (1 − 2u 2 )  + 1 + 2u 2  = 0  u 2 + 1 + 3u    ⇔ 1 − 2u 2 = 0 ⇔ 1 − 2( x + y ) = 0 2 x + 2 y = 1 2013 998 Sញ;ន  ⇔ x= ;y=− 30 x − 4 y = 2011 34 17 2013 998 .*ច ន U ប " បព< នQគM x = ;y=− 34 17 . =ង x = abc 'ចនន).5 e# ក 100a + 10b + c = 11(a 2 + b 2 + c 2 ) (1)  0 ≤ a; b; c ≤ 9 យ/ង;ន  a ≠ 0  a; b; c ∈ ℕ  យ/ង&ន (1) ⇔ 99a + 11b + ( a + c − b) = 11( a 2 + b 2 + c 2 ) ⇔ a + c − b = 11k ) −8 ≤ a + c − b ≤ 18 ⇒ k = 0 M k = 1 + k = 0 ⇒ a +c−b = 0 ⇒ b = a +c www.keoserey.wordpress.com Page 12
  • 14. បជុំវ គណិតវទ សិស ពូែកគណិតវទ ក់ទី១០ (1) ⇔ 9a + b = a 2 + b2 + c 2 ⇔ 9a + a + c = a 2 + (a + c)2 + c 2 ⇔ 10a + c = 2(a 2 + ac + c 2 ) ⇔ c = 2n (n ∈ ℕ) ⇔ 10a + 2n = 2( a 2 + 2na + 4n 2 ) ⇔ a 2 + (2n − 5) a + 4n 2 − n = 0 5កq:r ∆ ≥ 0 ⇒ − 12n 2 − 16n + 25 ≥ 0 −4 − 91 −4 + 91 ⇒ ≤n≤ ⇒ n=0 ⇒ c = 0; b = a នង a 2 − 5a = 0 6 6 ⇒ a = b = 5 នង c = 0 ⇒ x = 550 + k = 1 , យ/ង;ន a + c − b = 11 ⇒ b = a + c − 11 6 (1) ⇔ 99a + 11b + 11 = 11( a 2 + b 2 + c 2 ) ⇔ 9a + b + 1 = a 2 + b2 + c 2 ⇔ 9a + a + c − 11 + 1 = a 2 + (a + c − 11) 2 + c 2 ⇔ 10a + c − 10 = 2a 2 + 2c 2 + 121 + 2ac − 22a − 22c ⇔ 2a 2 + 2c 2 + 2ac − 32a + 23c + 131 = 0 ⇒ c 'ចនន ⇔ c = 2n + 1 . .*ច 6 យ/ង;ន a 2 + (2n − 15)a + 4n 2 − 19n + 55 = 0 យ/ង&ន ∆ = −12n 2 + 16n + 25 ≥ 0 4 − 91 4 + 91 ⇒ ≤n≤ ⇒ n ∈ {0; 1; 2} 6 6 • n = 2 ⇒ c = 5; b = a − 6 នង a 2 − 11a + 33 = 0 (AនU n ) • n = 1 ⇒ c = 3; b = a − 8 នង a 2 − 13a + 40 = 0 ⇒ a = 8; b = 0 នង c = 3 ⇒ x = 803 A • n = 0 ⇒ c = 1; b = a − 10 < 0 .*ច ន ប,ចនន).5 - e# កគM x = 550 នង x = 803 N K E O 1. + =ង H 'ចនច ប ពBទព ប " BE នDង ងBង" ( ABC ) T F K 'ចនច ប ពBទព ប " CF នDង ងBង" ( ABC ) I C យ/ង;ន AD = CK នង AD = BH B D P ⇒ BKHC 'ច :Zយ ម; M www.keoserey.wordpress.com Page 13
  • 15. បជុំវ គណិតវទ សិស ពូែកគណិតវទ ក់ទី១០ ⇒ TB = TK ⇒ TB + TC = TK + TC = CK = AD . + យ/ង&ន TB = NA ⇒ ND = TC ) ND = TM ⇒ TC = TM ⇒ ∆TMC ម; ង" T 1800 − MTC 1800 − AFT ⇒ TCM = = = OFC 2 2 ⇒ CM || OF . ⇒ M 4 2 5/ប6 " (∆ ) 7 " =ម C O/យ)កងនD ង AC 2នDង ⇒ P ច5< 2 5/ (∆ ) 2នD ង + យ I 4 2 5/ ម.uទ< ប " OP 6 IO = IP ) IP = d ( I ; ∆ ) ( Z IP )កងនDង (∆ ) ) Sញ;ន I ) ង 4 2 5/;9^9 ប* 5).5&នកន'ចនច O O/យប6 " ;ប" ទ 7 គM (∆ ) , ព5 D ច5< 2 5/ធ* *ច BC 2 1 1 E. យ x, y , z #ជ%&ន 6 យ/ង;ន 1 ≥ + > ⇒ y > 1 ⇒ y ∈ (1; 2) x y y 2 1 y −1 2y 2 ≤1− = ⇒ x≥ = 2+ x y y y −1 y −1 យ/ង&ន 4 4 ≤2− y ⇒ z ≥ z 2− y 2 4 .*ច 6 P ( x, y , z ) ≥ + + 9y + 2 y −1 2 − y  1  4 = 2 + 9( y − 1)  + + 9(2 − y ) + 2 ≥ 26  y −1  2− y 2y 4 1 I " = " ក/ &ន ⇔ x = w ;z= ; = 9( y − 1) y −1 2 − y y −1 4 4 នង = 9(2 − y ) ⇔ x = 8, y = នង z = 6 2− y 3 K. + យ/ង យបIក" Lemma % ង ម " កង 5 ចននគ ",កL យ, ) ង&នបចនន).5&នផ5ប*ក)ចក ច" នDង 3 " ^យ www.keoserey.wordpress.com Page 14
  • 16. បជុំវ គណិតវទ សិស ពូែកគណិតវទ ក់ទី១០ ក :ទមយ កង 5 ចននគ " ង 5/, &នបចនន k/ង ).5&ន :5" មA ព5 )ចកនDង 3 6 Lemma ង 5/គMព ក :ទព កង 5 ចននគ " ង 5/, &ន ច'ងបចនន).5&ន :5" មA ព5 )ចកនDង 3 6 នD ង&នបចនន).5&ន :5" @ងAគM 0;1; 2 ព5)ចកនDង 3 ព5 6 ផ5ប* កCនបចនន ន )ចក ច" នDង 3 Sញ;ន Lemma ព .*ច ន Lemma e#;ន យបIក" % + Fន# -នj Lemma ង 5/ យ/ង;ន • យក 5 កង 17 ចននគ ").5 ! 6 កង 5 ចនន).5 e#;នយក &នបចនន).5 &នផ5ប*ក)ចក ច" នDង 3 =ង បព<នQបចនន ន យ (a1 ; b1 ; c1 ) នង=ង m1 = a1 + b1 + c1 • យក 5 កង 14 ចននគ " 2 5" 6 កង 5 ចនន).5 e#;នយក &នបចនន).5 &នផ5ប*ក)ចក ច" នDង 3 =ង បព<នQបចនន ន យ (a2 ; b2 ; c2 ) នង=ង m2 = a2 + b2 + c2 • យក 5 កង 11 ចននគ " 2 5" 6 កង 5 ចនន).5 e#;នយក ន &នបចនន ).5&នផ5ប* ក)ចក ច" នDង 3 =ង បព<នQបចនន ន យ (a3 ; b3 ; c3 ) នង =ង m3 = a3 + b3 + c3 • យក 5 កង 8 ចននគ " 2 5" 6 កង 5 ចនន).5 e#;នយក ន &នបចនន ).5&នផ5ប* ក)ចក ច" នDង 3 =ង បព< នQបចនន ន យ (a4 ; b4 ; c4 ) នង=ង m4 = a4 + b4 + c4 • កង 5 ចននគ " 2 5", &នបចនន).5&នផ5ប* ក)ចក ច" នDង 3 =ង បព< នQ បចនន ន យ (a5 ; b5 ; c5 ) នង=ង m5 = a5 + b5 + c5 កង 5 ចននគ " m1; m2 ; m3 ; m4 ; m5 &នបចនន).5&នផ5ប* ក)ចក ច" នDង 3 zប&J បចនន 6 គM mi ; m j ; mk ព5 6 9 ចនន a1 ; bi ; ci ; a j ; b j ; c j ; ak ; bk ; ck &នផ5ប*ក)ចក ច" នDង 9 (បIg e#;ន យបIក") % www.keoserey.wordpress.com Page 15
  • 17. បជុំវ គណិតវទ សិស ពូែកគណិតវទ ក់ទី១០ N. យ/ងbច ជ/ / បព<នQក*F ន Oxy 89 ង, ! A, B , C &នក* F ន A(0; 0), B(a; b), C (c; d ) ).5 a; b; c; d 'ប,ចននគ " - zប&J D( x, y ) 'ផa ប " ងBង" ( ABC ), D 'ចនច&នក*F ន'ចននគ " យ/ង;ន AD 2 = BD 2 ⇒ x 2 + y 2 = ( x − a ) 2 + ( y − b) 2 ⇒ a 2 + b 2 − 2ax − 2by = 0 .*ច 6 a 2 + b 2 'ចននគ* ⇒ a, b &ន5កq: គ* .* ចA ⇒ a + b នង a − b 'ចននគ* .*ចA). , c + d , c − d 'ចននគ* ab ( a + b ) 2 − ( a − b ) 2 ⇒ = 'ចននគ " 2 4  a +b a −b c+d c−d  .*ច ន ប,ចនច X =  - ;  នង Y =  ;  'ប,ចនច&ន -  2 2   2 2  ក*F ន'ចននគ " ( a + b) 2 ( a − b) 2 a 2 + b 2 AB 2 AC 2 ) AX 2 = + = = , .* ចA). យ/ង&ន AY 2 = 4 4 2 2 2 2 2  a+b−c−d   a−b−c+ d  XY =  2  +   2   2  1 ( (b − c) + (a − d ) ) + ( (a + d ) − (b + c) )  2 2 = 4  1 = (b − c)2 + (a − d )2 + 2(b − c)(a − d ) + (a + d )2 + (b + c)2 − 2(a + d )(b + c)  4  1 =  2a 2 + 2d 2 + 2b 2 + 2c 2 + 2ab − 2ac − 2bd + 2cd − 2a − 2ac − 2bd − 2cd  4  1 2 = 2a + 2b 2 + 2c 2 + 2d 2 − 4ac − 4bd  4   (a − c) 2 + (b − d ) 2 BC 2 = = 2 2 6 ! ∆AXY &ន^ង.*ចនD ង ∆ABC នង AXY *ច'ង ∆ABC , ផ7យពប^ប" .*ច ន D មនbច'ចនច).5&នក*F ន'ចននគ " ទ '()& www.keoserey.wordpress.com Page 16
  • 18. បជុំវ គណិតវទ សិស ពូែកគណិតវទ ក់ទី១០ វ1 េសរ បឡងអូ ពិចេវ ត ំ ម*ក់ទី១០ េលកទី XVII វ ទី៣  x 2 + 4 y 2 − 4 x + 12 y + 11 = 0  . យ បព<នQ ម ង ម  2  x + 4 y − 2 xy − x + 4 y − 12 = 0 2  . កU 'ចននគ " ប " ម x1 + x2 + ... + x15 = 1215 4 4 4 1. : ABC ផ7GងH "5កq:FB ប/ ? ផ7GងH " 5កq: 7 r 7 r ង ម cos A cos B cos C 17 + + = 8 15 17 120 E. គ ! a, b, c 'បចននព #ជ%&ន ផ7GងH " abc = 1 7 បfញJ g  1 1 1  1 1 1  9  + +  + + ≥  a b c 1+ a 1+ b 1+ c  2 K. កង ន T = {1, 2, ..., 2010} /&នប9 6នចនន).5)ចកមន n ច" នDង 2,3,5, 7,11 ? N. គ !ច :)កង ABCD &ន M 'ចនចក,5 ប - " AB , N 4 2 5/កន3 ប6 "ព ប 7 " BCD =ង P 'ច ,5)កង ប " N 2 5/ BC យបIក"J ប/ MN ⊥ DP 6 % : AND ម; '()& ចំេលយ . បព<នQ).5 ! មម* 5នDង  x 2 + 4 y 2 − 4 x + 12 y + 11 = 0 (1)  x 2 + 4 y 2 − 4 x + 12 y + 11 = 0    ⇔  3x − 23 (2 x + 8) y = 3x − 23  y = (2)  2x + 8 ជន (2) ច*5 (1) យ/ង;ន 2  3x − 23  3x − 23 x + 4 2  − 4 x + 12. + 11 = 0  2x + 8  2x + 8 ⇔ 4 x 4 + 16 x3 + 88 x 2 − 720 x + 620 = 0 x =1 ⇔ ( x − 1)( x − 4)(4 x 2 + 32 x + 204) = 0 ⇔  x = 4 www.keoserey.wordpress.com Page 17
  • 19. បជុំវ គណិតវទ សិស ពូែកគណិតវទ ក់ទី១០ + ច Z x =1 ⇒ y = −2 +ចZ x = 3 ⇒ y = −1 .*ច ន ម &នច 5/យព (1; −2), (3; −1) . កU 'ចននគ " ប " ម x1 + x2 + ... + x15 = 1215 4 4 4 +ចZ xi = 2k : xi4 = 16k 4 ≡ 0 (mod 16) +ចZ xi = 2k + 1 , យ/ង;ន xi4 − 1 = ( xi − 1)( xi + 1)( xi2 + 1) = 4k ( k + 1)( xi2 + 1) យ k ( k + 1) ⋮ 2; xi2 + 1 ⋮ 2 ( Z xi ) 6 xi4 − 1⋮ 16 M xi4 ≡ 1 (mod 16) 15 យ/ង;ន ∑ xi4 ≡ r (mod 16) កង 6 0 ≤ r ≤ 15 i =1 មu9 ង ទ@ 1215 = 75.16 + 15 ≡ 15 (mod 16) , .*ច 6 xi ច Z គប" i = 1,15 O/យ យ 7 4 = 2401 > 1215 6 xi ≤ 5 ∀i យ 54 + 54 = 1250 > 1215 6 &ន ច/នបផ ចនន xi មយ, zប&JគM x15 , ផ7GងH " x15 = 54 = 625 7 4 ព5 6 x1 + x2 + ... + x14 = 1215 − 625 = 590 នង xi ≤ 3 ∀i = 1,14 , 4 4 4 ) 590 = 34.7 + 23 6 e#&ន ច/នបផ 7 ចននកង 14 ចនន x1 , x2 , ..., x14 ផ7Gង H " | xi |= 3 , ) 7 590 < 34.8 , មន ម O ផ5 .*ច ន | xi | ≤ 3 ច Z គប" i = 1,15 យ/ង;ន 1215 = x1 + x2 + ... + x15 ≤ 15.34 = 1215 , .* ច 6 | xi | = 3 គប" i = 1,15 4 4 4 .*ច ន ម ).5 !&ន 215 U 'ចននគ " &ន^ង ( ±3; ± 3; ... ; ±3) cos A cos B cos C 15.17 cos A + 8.17 cos B + 8.15cos C 1. + + = 8 15 17 2040 82 + 152 + 17 2 17 ≤ = 4080 120 ព '.*ច ន 82 − 2.8.(17 cos B + 15cos C ) + 152 + 17 2 − 2.15.17 cos A = [8 − (17 cos B + 15 cos C ) ] + 152 + 17 2 − 2.15.17 cos A − (17 cos B + 15 cos C ) 2 2 www.keoserey.wordpress.com Page 18
  • 20. បជុំវ គណិតវទ សិស ពូែកគណិតវទ ក់ទី១០ ) 152 + 17 2 − 2.15.17 cos A − (17 cos B + 15cos C )2 = 152 sin 2 C + 17 2 sin 2 B − 2.15.17(cos A + cos B cos C ) = 152 sin 2 C + 17 2 sin 2 B − 2.15.17 sin C sin B = (15sin C − 17 sin B ) 2 ≥ 0 cos A cos B cos C 17 .*ច ន + + ≤ 8 15 17 120 15sin C = 17 sin C មiព ក/ &ន5 =)  8 = 17 cos B + 15cos C  15 17  sin B = sin C  15 17 8 ⇔  ⇔ = = 8 = 15sin C cos B + 15cos C sin B sin C sin A   sin B 15 17 8 M = = b c a .*ច 6 : ABC &ន^ង.* ចនD ង :).5&ន ជmងSងប /n 8,15,17 យ 82 + 152 = 17 2 , 6 ! : ABC )កង ង" C  1 1 1  1 1 1  E. T =  + +  + +  = T1 + T2 + T3  a b c 1+ a 1+ b 1+ c  1 1 1 ).5 T1 = + + a (1 + a ) b(1 + b) c(1 + c ) 1 1 1 1 1 1 T2 = + + នង T3 = + + b(1 + a ) c(1 + b) a (1 + c) c(1 + a ) a (1 + b) b(1 + c) យ 6ទ a, b, c .* ចA 6 យ/ងbចzប&J a ≤ b ≤ c , ព5 6 1 1 1 1 1 1 ≥ ≥ នង ≥ ≥ a b c a +1 b +1 c +1 យ/ង;ន T1 ≥ T2 ព '.* ច ន 1 1 1  1 1 1  T1 − T2 = + + − + +  a (1 + a ) b(1 + b) c(1 + c)  b(1 + a ) c (1 + b) a (1 + c )   1 1 1  1 1 1  1 1 1 = −  + −  + −  1+ a 1+ c  a 1+ b 1+ a  b 1+ c 1+ b  c www.keoserey.wordpress.com Page 19
  • 21. បជុំវ គណិតវទ សិស ពូែកគណិតវទ ក់ទី១០  1 1  1 1   1 1  1 1  = −  −  +  −  −  ≥ 0  1 + a 1 + c  a b   1 + b 1 + c  b c  3 យ/ង;ន T2 ≥ ព '.* ច ន 2 y z x Z abc = 1 6 =ង a = ,b = ,c = ព5 6 x y z x y z  1 1 1  3 T2 = + + = ( x + y + z)  + + −3 ≥ y+z z+x x+ y  y+z z+x x+ y 2 3 .*ចA). T3 ≥ 2 9 .*ច ន T≥ , មiព ក/ &ន5 =) a = b = c =1 2 K. =ង A1 = {k ∈ T / k ⋮ 2} ; A2 = {k ∈ T / k ⋮ 3} ; A3 = {k ∈ T / k ⋮ 5} ; A4 = {k ∈ T / k ⋮ 7} ; A5 = {k ∈ T / k ⋮11} ; ព5 6 A1 ∪ A2 ∪ A3 ∪ A4 ∪ A5 ' នប,ចនន - 4 2កង T )ចក ច"នDងចនន មយកងប,ចនន 2,3,5, 7,11 - 2010 2010 2010 យ/ង;ន | A1 | = = 1005; | A2 | = = 670; | A3 | = = 402 2 3 5  2010   2010   2010  | A4 | =   = 287; | A5 | =  11  = 182; A1 ∩ A2 =  6  = 335;  7       2010   2010   2010  A1 ∩ A3 =   = 201; A1 ∩ A4 =  14  = 143; A1 ∩ A5 =  22  = 91;  10       2010   2010  A2 ∩ A3 =   = 134; A2 ∩ A4 =  21  = 95;  15     2010   2010  A2 ∩ A5 =   = 60; A3 ∩ A4 =  35  = 57;  33     2010   2010  A3 ∩ A5 =   = 36; A4 ∩ A5 =  35  = 57;  33     2010   2010  A1 ∩ A2 ∩ A3 =   = 67; A1 ∩ A2 ∩ A4 =  42  = 47;  30     2010   2010  A1 ∩ A2 ∩ A5 =   = 30; A1 ∩ A3 ∩ A4 =  70  = 28;  66    www.keoserey.wordpress.com Page 20
  • 22. បជុំវ គណិតវទ សិស ពូែកគណិតវទ ក់ទី១០  2010   2010  A1 ∩ A3 ∩ A5 =   = 18; A1 ∩ A4 ∩ A5 =  154  = 13;  110     2010   2010  A2 ∩ A3 ∩ A4 =   = 19; A2 ∩ A3 ∩ A5 =  165  = 12;  105     2010   2010  A2 ∩ A4 ∩ A5 =   = 8; A3 ∩ A4 ∩ A5 =  385  = 5;  231     2010   2010  A1 ∩ A2 ∩ A3 ∩ A4 =   = 9; A1 ∩ A2 ∩ A3 ∩ A5 =  330  = 6;  210     2010   2010  A1 ∩ A2 ∩ A4 ∩ A5 =   = 4; A1 ∩ A3 ∩ A4 ∩ A5 =  770  = 2;  462     2010   2010  A2 ∩ A3 ∩ A4 ∩ A5 =  = 1; A1 ∩ A2 ∩ A3 ∩ A4 ∩ A5 =  =0  1155    2310   5 ) យ A1 ∪ A2 ∪ A3 ∪ A4 ∪ A5 = ∑ Ai − ∑ Ai ∩ A j ∩ Ak − i =1 1≤i < j < k ≤5 − ∑ Ai ∩ A j ∩ Ak ∩ Aq + A1 ∩ A2 ∩ A3 ∩ A4 ∩ A5 = 1593 1≤i < j < k < q ≤5 .*ច ន ន T &ន 2010 − 1593 = 417 ចនន).5មន)ចក ច"នDង 2, 3, 5, 7,11 N. ជ/ / យ89 ង, ! A ≡ O; B ∈ Oy , D ∈ Ox  b ព5 6 A(0;0), B(0; b), D(d ;0), C (d , b) , M  0;  , N (t ; t + b − d ), P (t ; b)  2 MN ⊥ DP ⇔ MN .DP = 0  b  b2 ⇔ t (t − d ) + b  t + − d  = 0 ⇔ t 2 + (b − d )t + − bd = 0  2  2 AN 2 = t 2 + (t + b − d ) 2 , AD 2 = d 2 យ/ង;ន AN = AD ⇔ AN 2 = AD 2 ⇔ d 2 = t 2 + (t + b − d ) 2 b2 ⇔ t 2 + (b − d )t + − bd = 0 (បIg e#;ន យបIក") % 2 '()& www.keoserey.wordpress.com Page 21
  • 23. បជុំវ គណិតវទ សិស ពូែកគណិតវទ ក់ទី១០ វ1 េសរ បឡងអូ ពិចេវ ត ំ ម*ក់ទី១០ េលកទី XVII វ ទី៤  x + y + z = 1 (1)  . កU 'ចននព x; y; z ប " បព<នQ ម  x y z 9  x + yz + y + zx + z + xy = 4 (2)  . ក គប"ចននគ "#ជ%&ន p , q , n ច Z p, q 'ចននប[ម ./ម0 ! p ( p + 3) + q (q + 3) = n(n + 3) 1. កងប3ង" គ !ចនច P 2នD ង ពន ! :)កង ម; ABC (&ន ABC = 900 ; AB < 5 ) ផ7GងH "5កq: PA = 2 នង PB = 3 7 r ក C5ធបផ ប "Fង` " PC E. គ !ប,ចននព - a; b; c ផ7GងH " 7 0 < a ≤ b ≤ c; c ≥ 9; 8c ≥ 36 + bc;12c ≥ 36 + bc + 4ac យបIក"J % a + b− c ≤0 , មiព ក/ &ន 2 ព5,? K. កង នប,ចននគ " ធមn' ព 2 .5" 2011 , គ ជ/ - យក 1006 ចនន O/យប ង`/ ' ន ង A = {a1 ; a2 ; ... ; a1006 } យបIក"J កង A &នយក;ន 2 ចនន ./ ម0 !ចននមយ ន )ចក % ច" នDងចនន មយ ទ@ N. គ ! 6 ចននព a; b; c; d ; e; f ) ប ប|5 ផ7GងH "5កq: 7 r a 2 + b 2 − 2a = c 2 + d 2 − 2c = 3; e 2 + f 2 − 10e + 6 f + 33 = 0 a 2 + c 2 + b 2 + d 2 − 2ac − 2bd = 12; នង (e − 5)(a + c − 2e) + ( f + 3)(b + d − 2 f ) = 0 2 2 a+c  b+d  ក: " C5 *ចបផ ប "ក នyម S =  − e +  −f  2   2  '()& www.keoserey.wordpress.com Page 22
  • 24. បជុំវ គណិតវទ សិស ពូែកគណិតវទ ក់ទី១០ ចំេលយ . យ x; y; z #ជ%&ន 6 xy xz yz yx zx zy x + y + z =1 ⇔ + + = 1 (*) z y x z y x A yz B zx C xy =ង tan = ; tan = ; tan = ).5 0 < A; B; C < π (**) 2 x 2 y 2 2 A B B C C A ម (1) bច k/ង#ញ' tan tan + tan tan + tan tan = 1 (3) 2 2 2 2 2 2 1 1 1 9 O/យ ម (2) bច ' + + = (4) A B C 4 1 + tan 2 1 + tan 2 1 + tan 2 2 2 2 A B C B C A B C (3) ⇔ tan  tan + tan  = 1 − tan tan ⇔ tan = cot  +  2 2 2 2 2 2 2 2 ⇔ A + B + C = π + k 2π , =ម (**) 6 A + B + C = π ព5 6 A B C 9 3 + cos A + cos B + cos C 9 (4) ⇔ cos 2 + cos 2 + cos 2 = ⇔ = 2 2 2 4 2 4 3 A A B−C 3 ⇔ cos A + cos B + cos C = ⇔ 1 − 2sin 2 + 2sin cos = 2 2 2 2 2 A A B−C ⇔ 4sin 2 − 4sin cos +1 = 0 2 2 2  A B−C 2sin 2 = cos 2  π ⇔  ⇔ A= B =C = sin B − C = 0 3   2 1 បព<នQ ម &នច 5/យ x = y = z = 3 . ច Z m 'ចននគ "#ជ%&ន, យ/ង ក;ន :5" ព5)ចក m( m + 3) នDង 3 ពន ! 3 ក : m = 3k 6 m(m + 3) ≡ 0 (mod 3) www.keoserey.wordpress.com Page 23
  • 25. បជុំវ គណិតវទ សិស ពូែកគណិតវទ ក់ទី១០ m = 3k + 1 6 m(m + 3) ≡ 1 (mod 3) m = 3k + 2 6 m(m + 3) ≡ 1 (mod 3) .*ច ន ព5 m )ចក ច" នDង 3 6 m(m + 3) )ចក ច" នDង 3 , ព5 m )ចកមន ច" នDង 3 6 m(m + 3) )ចកនDង 3 ;ន :5" /n 1 kប"មក5l " យ/ង#ញ, យ/ង&ន p( p + 3) + q (q + 3) ≡ n(n + 3) (mod 3) + ក : : ប/ p នង q ទQ) )ចកមន ច" នDង 3 6 p( p + 3) + q(q + 3) ≡ 2 (mod 3) . កង ព5 6 n(n + 3) ≡ 0 (mod 3) M n(n + 3) ≡ 1 (mod 3) .*ច ន ក : ន មន ផ7GងH " 7 + ក : : កងព ចនន p, q &នមយ)ចក ច" នDង 3 zប&J p )ចក ច" នDង 3 , ) p 'ចននប[ម 6 p=3 ព5 6 18 + q (q + 3) = n(n + 3) , Sញ;ន 18 = n(n + 3) − q(q + 3) (*) យ n > q 6 n(n + 3) ≥ q(q + 1)(q + 4) (**) ព (*) & (**) យ/ង;ន 18 ≥ 2q + 4 ⇒ q ≤ 7 .*ច ន q ∈{2; 3; 5; 7} ( Z q 'ចននប[ម) ច Z q = 2 6 18 + 10 = n(n + 3) ⇒ n = 4 ច Z q = 3 6 18 + 18 = n(n + 3) ⇒ មន&ន n ច Z q = 5 6 18 + 40 = n(n + 3) ⇒ មន&ន n ច Z q = 7 6 18 + 70 = n(n + 3) ⇒ n = 8 ពន !.*ចA). ព5 q = 3 p =3 p =3 p = 2 p = 7     .*ច ន យ/ង;នប,U - គM q = 2 ; q = 7 ;  q = 3 ; q = 3 n = 4 n = 8  n = 4 n = 8     1. យ/ង&ន PC )#ងបផ ព5 P នង C 2&ង&ក" ធ@បនD ងប6 " AB q 7 ង" :)កង ម; APQ ( AP = PQ; Q នង B 4 2 ង) មយ ធ@បនDង AP ) , www.keoserey.wordpress.com Page 24
  • 26. បជុំវ គណិតវទ សិស ពូែកគណិតវទ ក់ទី១០ 0 45 ចពន !ចtប" បងB5 QA (ផa A; ម ងB5 450 ) : P ֏ P ' នង B ֏ B ' ពន !ចtប" ប)5ងង V A 2 (ផa A , ផ5 ធ@ប k = 2 ) : P ' ֏ Q នង B ' ֏ C យ/ង;ន PQ = AP = 2 នង QC = 2 P ' B ' = 2 PB = 3 2 យ PC ≤ PQ + QC ⇒ PC ≤ 2 + 3 2 .*ច ន max PC = 2 + 3 2 ទទ5;ន ព5 P 4 2 5/ប6 " PQ 7 ⇔ AC = AP 2 + PC 2 = 4 + (2 + 3 2)2 = 26 + 12 2 9 c ≤1  b 9 E. =មប^ប"Sញ;ន  + ≤2 (*)  4 c  b 9 a + 4 + c ≤ 3   9  b 9 ប)5ង a + b + 9 = a +  b 4 + +   c  4 + +  c ( c −2 ) 9 c  Fន# -នj# មiព Bunhiakovski យ/ង;ន b 9 b 9 b 9  b 9 + ≤ 2  +  នង a+ + ≤ 3 a + +  4 c 4 3 4 c  4 3 ប/ ; " (*) Sញ;ន a + b + 9 ≤ 3+ 2+ c − 2 = 3+ c ⇒ a + b− c ≤0 ព , មiព ក/ &ន ព5 a = 1; b = 4; c = 9 K. + ប)5ង a1 = m1b1; a2 = m2b2 ; ... ; ai = m1006b1006 , កង 6 mi ; bi 'ចននគ "#ជ%&ន, bi 'ចនន &ន C5ព 2 P.5" 2011 , (ច Z i = 1; 2; ...; 1006 ) + ព 2 P.5" 2011 &ន 1005 ចនន ).5 e# ជ/ យក 1006 ចនន bi + =ម ទD -បទ Dirichlet 6 កងប,ចនន bi ).5 ទ/ប ជ/ - យក&នព ចននគM b j = bk ⇒ a j = 2m j b j ; ak = mk bk (*) + zប&J a j > ak (**) + ព (*) នង (**) យ/ង;ន a j )ចក ច" នDង ak www.keoserey.wordpress.com Page 25
  • 27. បជុំវ គណិតវទ សិស ពូែកគណិតវទ ក់ទី១០ N. កងប3ង"ក: " យ បព<នQក*F ន. Oxy , ពន ! ងBង" (C1 ) ផa I1 (1;0) នង R1 = 2 , O/យ ងBង" (C2 ) ផa I 2 (5; −3) R2 = 1 ពប^ប" យ/ង;ន A(a; b), B(c; d ) 4 2 5/ (C1 ) នង D(e; f ) 4 2 5/ (C2 ) O/យ AB = 2 3 a+c b+d  d E 'ចនចក,5 ប - "Fង` " AB យ/ង;ន E  ;   2 2  2 ពប^ប" យ/ង;ន I 2 D.2 DE = 0 , ព5 6 S = ED = EI 2 − R2 = EI 2 − 1 2 2 2 2 2  AB  =ង (C3 ) ' ងBង"ផa I1 (1;0) R3 = I1 E = R1 −   = 4 − 3 =1  2  យ/ង;ន E " 2 5/ ងBង" (C3 ) =ង H ; K 'ប,ចនច ប - ពB ប "ប6 " I1I 2 នDង (C3 ) ( HI 2 > KI 2 ) 7 យ/ង;ន S ≥ KI 2 − 1 = ( I1 I 2 − R3 ) 2 − 1 = (5 − 1) 2 − 1 = 15 2 min S = 15 ទទ5;ន ព5 E ≡ K O/យ D 'ចនច ប ពB ប " (C2 ) នDង ងBង" (C ') &នFង` "ផa KI 2 យ ម ប6 " I1I 2 គM 3x + 4 y − 3 = 0 O/យ ងBង" (C3 ) : ( x − 1) 2 + y 2 = 1 7 9 3 a+c 9 b+d 3 ⇒ K  ;−  ⇒ = ; =− 5 5 2 5 2 5  9+3 3 −3 + 4 3 9−3 3 −3 − 4 3 a = ;b = ;c= ;d = 5 5 5 5 ⇒   9+3 3 −3 + 4 3 9−3 3 −3 − 4 3 c = ;d = ;a= ;b =  5 5 5 5 2 2  17   9 យ ម ងBង" (C2 ) : ( x − 5) + ( y + 3) = 1 នង ងBង" (C ') :  x −  +  y +  = 4 2 2  5  5  24 9 5 3 171   24 9 5 3 171  ⇒ D + ; −  5 20 3 5 60  M D −  5 20 3 ;− −     5 60  www.keoserey.wordpress.com Page 26
  • 28. បជុំវ គណិតវទ សិស ពូែកគណិតវទ ក់ទី១០  24 9 5 3 171 e = + ;f = − 5 20 3 5 60 ⇒   24 9 5 3 171 e = − ; f =− −  5 20 3 5 60 '()& វ1 េសរ បឡងអូ ពិចេវ ត ំ ម*ក់ទី១០ េលកទី XVII វ ទី៥ (3 x + y )( x + 3 y ) xy = 14  . យ បព<នQ ម  ( x + y )( x + 14 xy + y ) = 36 2 2  . ក គប"ប,ចននប[ម p ./ ម0 ! 211 p − 2 )ចក - ច"នDង 11p 1. គ ! : ABC &នប, ម.uន AA , BB1 , CC1 ប - 1 ពBA ង" ចនច G ( A1 , B1 , C1 4 2 5/ប, ជmង ប - " : ABC ) ប6យ AA , BB1 , CC1 3 1 " ងBង" Dក ] : ABC =ម5 ប" ង" A2 , B2 , C2 GA2 GB2 GC2 យបIក"J % + + ≥3 GA GB GC 2 (a + b + c)3  ab + bc + ca  E. គ ! a, b, c 'ប,ចននព #ជ%&ន បfញJ - g + 2  ≥ 28 abc  a + b2 + c2  K. គ ! 1007 ចនច ផpងA 2 5/ប3ង" យបIក" J &ន89 ង ច 2011 ចនចក,5 % - ផpងAពប,គ* ចនច ន , - / ព5,).5&ន 2011 ចនចក,5? - N. ប6 "ព កង នងព 7 ]ប "ម C ប " : ABC "ប6 " AB 7 ង" E នង D យបភ3J ប/ CE = CD 6 AC 2 + BC 2 = 4 R 2 ( R ' ងBង" ក ] D : ABC ) '()& www.keoserey.wordpress.com Page 27
  • 29. បជុំវ គណិតវទ សិស ពូែកគណិតវទ ក់ទី១០ ចំេលយ (3 x + y )( x + 3 y ) xy = 14 (1)  . យ បព<នQ ម  ( x + y )( x + 14 xy + y ) = 36 (2) 2 2  =ង x = u ≥ 0; y = v ≥ 0 , យ/ង;ន ( uv 3u 4 + 10u 2 v 2 + 3v 4 = 14  (1), (2) ⇔  ) u 6 + 15u 4 v 2 + 15u 2 v 4 + v 6 = 36  3u 5v + 10u 3v3 + 3uv5 = 14  ⇔  u + 15u v + 15u v + v = 36 6 4 2 2 4 6  36 + 2.14 = u 6 + 6u 5v + 15u 4v 2 + 20u 3v3 + 15u 2v 4 + 6uv5 + v 6  ⇔  36 − 2.14 + u − 6u v + 15u v − 20u v + 15u v − 6uv + v 6 5 4 2 3 3 2 4 5 6   (u + v) = 64 = 2 6 6 u + v = 2 u + v = 2  ⇔  ⇔  ∨  ( ) 6 (u − v) = 8 = 2 6 u − v = 2 u − v = − 2   យ (u , v ≥ 0 )  2  2 u = 1 + u = 1 −  2  2 ⇔  M  v = 1 − 2 v = 1 + 2   2   2 3 3  3 3  .*ច ន បព<នQ&នច 5/យ  + 2; − 2  នង  − 2; + 2  2 2  2 2  . zប&J p 'ចនន).5 e# ក, យ/ង;ន 211 p ≡ 2 (mod p ) (1) Fន# -នj ទD បទ Fermat ច Z ចននប[ម p , យ/ង;ន - 211 p ≡ 211 (mod p ) (2) ព (1) & (2) Sញ;ន p ' )ចក ប " 211 − 2 = 2046 = 2.3.11.31 ) p 'ចននប[ម 6 p ∈ {2, 3,11,31} + ពន ! p = 2 យ/ង&ន 222 ≡ 2 (mod 22) ,Sញ;ន 221 ≡ 1 (mod 11) (3) មu9 ង ទ@ 210 ≡ 1 (mod 11) (4) www.keoserey.wordpress.com Page 28
  • 30. បជុំវ គណិតវទ សិស ពូែកគណិតវទ ក់ទី១០ ( ) 2 6 220 = 210 ≡ 1 (mod 11) , Sញ;ន 221 ≡ 2 (mod 11) , ផ7យនDង (3) .*ច ន p=2 e# 5 + ពន ! p = 3 ( ) .22 ≡ 4 (mod 11) , .*ច 6 3 ព (4) យ/ង;ន 232 = 220 232 ≡ 1 (mod 11) / Sញ;ន 232 ≡ 1 (mod 33) នង 232 ≡ 2 (mod 33) / / .*ច ន p=3 e# 5 + ពន ! p = 11   ( ) 4 យ/ង&ន 211 p − 2 = 2121 − 2 = 2  230 − 1   = 2((230 )2 + 1)(230 + 1)(25 + 1)(210 − 25 + 1)(25 − 1)(210 + 25 + 1) . ជន 25 យ 32 O/យគបផpនD ង (4) , Sញ;នកងផ5គ: ន &ន) ក=គ: - 25 + 1 ទ).5)ចកនDង 11 ;ន 3 , •ប,ក=គ: ផpង ទ@ - - ទQ) )ចកមន ច" នDង 11 , 6 ផ5គ:)ចកមន ច" នDង 11 p = 121 .*ច ន p = 11 e# 5 + ពន ! p = 31 យ/ង e#ពន ! ម/5 2340 ≡ 1 (mod 341) M ទ 34 ព (4) Sញ;ន 2340 = 210 ≡ 1 (mod 11) (5) មu9 ង ទ@ 25 ≡ 1 (mod 31) 6 2340 = (25 )68 ≡ 1 (mod 31) គបផpចនច ន នD ង (5) Sញ;នបIg e#ពន !គMព ( 11 នង 31 ទQ) 'ប,ចននប[ម) - .*ច ន ចននប[ម).5 e# កគM p = 31 a2 a2 1. យ/ង&ន AA1. A1 A2 = A1 B. A1C = ⇒ A1 A2 = 4 4ma 1 a2 a 3 GA2 a 3 មu9 ង ទ@ GA2 = GA1 + A1 A2 = ma + ≥ 6 ≥ 3 4ma 3 GA 2ma www.keoserey.wordpress.com Page 29